6 Applications of Trigonometry in 3-Dimensional Problems: Review Exercise 6 (P. 6.5)

Download as doc, pdf, or txt
Download as doc, pdf, or txt
You are on page 1of 66
At a glance
Powered by AI
The key takeaways are applications of trigonometry to solve problems in 3-dimensional geometry, such as calculating areas, angles between lines/planes, and bond angles in molecules.

To calculate the area of a triangle using trigonometry, you use the formula Area = 1/2 * a * b * sin(C) where a and b are two sides of the triangle and C is the angle between them.

To calculate the angle between two planes, you find the angle between two intersecting lines where one line lies in each plane.

NSS Mathematics in Action (2nd Edition) 5A Full Solutions

A  B  C  180 ( sum of △ )
6 Applications of
C  180  100.8625  45
Trigonometry in 3-  34.1 (cor. to 3 sig. fig.)
dimensional Problems

Review Exercise 6 (p. 6.5)


1
 (5)(8) sin 95 cm 2
1. Area of △ABC 2
 19.9 cm 2 (cor. to 3 sig. fig.)

AB  BC  CA
2. Let s  .
2
4  9  10
∴ s cm
2
 11.5 cm
∴ Area of △ABC
 s ( s  AB)( s  BC )( s  CA)
 11 .5(11 .5  4)(11 .5  9)(11 .5  10) cm 2
 18.0 cm 2 (cor. to 3 sig. fig.)

3. BAC  55  45  180 ( sum of △)


BAC  80
By the sine formula,

AB BC

sin C sin A
20 cm a

sin 55 sin 80
a  24.0 cm (cor. to 3 sig. fig.)

4. By the cosine formula,

PQ 2  QR 2  PR 2
cos∠ PQR 
2( PQ)(QR)
7 2  62  52
cos 
2(7)(6)
  44.4 (cor. to 3 sig. fig.)

5. By the cosine formula,

b 2  a 2  c 2  2ac cos B
b 7 2  4 2  2(7)( 4) cos 45 m
 5.0400 m
 5.04 m (cor. to 3 sig. fig.)

b2  c 2  a 2
cos A 
2bc
5.0400 2  42  7 2

2(5.0400)(4)
A  100.8625
 101 (cor. to 3 sig. fig.)

204
6 Applications of Trigonometry in 3-dimensional Problems

6. By the cosine formula,

b2  c2  a2
cos A 
2bc
82  122  10 2

2(8)(12)
A  55.7711  8. (a) With the notations in the figure,
 55.8 (cor. to 3 sig. fig.)
a2  c2  b2
cos B 
2ac
10  12 2  8 2


2(10)(12)
B  41.4096
 41.4 (cor. to 3 sig. fig.)
A  B  C  180 ( sum of △)
C  180  55.7711  41.4096 ∠ PAR  ∠ PAB  180 (adj. s on st. line)
 82.8 (cor. to 3 sig. fig.) 65  ∠ PAB  180
∠ PAB  115 
7. (a) With the notations in the figure, In △PAB, by the sine formula,

sin∠ ABP sin∠ PAB



AP BP
sin∠ ABP sin 115 

20 km 29 km
20 sin 115 
sin∠ ABP 
29
∠ ABP  38.6851 or 180 – 38.6851
BAQ  ABP (alt. s, QA // BP)
 38.7 (cor. to 3 sig. fig.)
 32
The compass bearing of B from A
or 141 (rejected)
∴ The compass bearing of P from B is N38.7°E.
 S(90  32) W
 S58 W
(b) ∠ ABP  ∠ BPT  38.6851 (alt.∠s, AB // PT)
∴ The true bearing of B from P is (180° + 38.6851°),
(b) With the notations in the figure, i.e. 219° (cor. to 3 sig. fig.).

9. With the notations in the figure,

RCB  PBC (alt. s, RC // BP)


 18
ACB  180  70  (32  18) (
 60
sum of △) ∠ HPR  ∠ PHS  62 (alt.∠s, PR // SH)
The true bearing of A from C In △HPR,

 270  RCB  ACB


 270  18  60
 348

205
NSS Mathematics in Action (2nd Edition) 5A Full Solutions

HR By the cosine formula,


sin∠ HPR  HQ 2  PQ 2  HP 2
HP cos∠ HQP 
80 m 2( HQ)( PQ )
sin 62 
HP 95.7085 2  10 2  90.6056 2

80 2(95.7085)(10)
HP  m
sin 62 ∠ HQP  56.7 (cor. to 3 sig. fig.)
 90.6056 m ∴ The angle of elevation of H from the man at Q is 56.7°.
∠ HPQ  ∠ HPR  180
(adj.∠s on st. line)
∠ HPQ  62  180 Activity
∠ HPQ  118 
In △HPQ, by the cosine formula, Activity 6.1 (p. 6.12)
1. (a) yes
HQ 2  PQ 2  HP 2  2( PQ )( HP) cos∠ HPQ (b) YP
HQ  10  90.6056  2(10)(90.6056) cos118  m
2 2

FD XY
 95.7085 m (c) Slope of PF  , slope of PX  ,
DP YP
EC
slope of PE 
CP

2. PX. Since FD = XY = EC and YP is the shortest in length


among the line segments DP, CP and YP, the value of
XY
is the greatest. PX has the greatest slope.
YP

Maths Dialogue

Maths Dialogue (p. 6.22)


1. yes

2. yes.
Let X and Y be the mid-points of CD and BE respectively.
∵ AC  AD and CX  XD
∴ AX  CD (prop. of isos. △)
∵ DEYX is a rectangle.
∴ YX  CD
∵ AX  CD and YX  CD
∴ The angle between planes  3 and  4 is ∠AXY.

Classwork

Classwork (p. 6.9)


(a) 
(b) 
(c) 
(d) 
(e) 
(f) 

Quick Practice

Quick Practice 6.1 (p. 6.8)


(a) The angle between the lines AB and BC is∠ABC.

(b) The angle between the lines DF and EF is∠DFE.

206
6 Applications of Trigonometry in 3-dimensional Problems

(c) Join AF. MC


tan∠ CBM 
BC
1
 8 cm
 2
8 cm
∠ CBM  26.6 (cor. to 3 sig. fig.)
∴ The angle beteween the planes MAB and ABCD is
26.6°.
∵ AF and EF intersect at F.
∴ The angle between the lines AF and EF is∠AFE. Quick Practice 6.4 (p. 6.14)
∵ BF is the line of greatest slope of the plane ABFE.
Quick Practice 6.2 (p. 6.10) ∴ ∠CBF is the inclination of the line of greatest slope of the
(a) Join CE and CA. plane ABFE.
EF = AB = 32 cm
Consider △BEF.
BF
tan∠ BEF 
EF
BF
tan 25 
32 cm
∵ CA is the projection of CE on the plane ABCD. BF  32 tan 25 cm
∴ The angle between the line CE and the plane ABCD is Consider △BCF.
∠ACE. BC
cos∠ CBF 
(b) Consider △ABC. BF
∵ ABCDHEFG is a cube. 10 cm
∴ ABCD is a square. 
i.e. ∠ABC = 90°
32 tan 25 cm
∠ CBF  47.9 (cor. to 3 sig. fig.)
AC  AB 2  BC 2 (Pyth. theorem)
∴ The inclination of the line of greatest slope of the plane
 6 6
2 2
cm ABFE to the horizontal is 47.9°.
 72 cm
Quick Practice 6.5 (p. 6.16)
Consider △ACE. (a) ∵ N is a point on CD and the distance between N and the
AE horizontal plane is 23 cm.
tan∠ ACE  ∴ CE = 23 cm
AC
Consider △BCE.
6 cm CE
 tan∠ CBE 
72 cm BE
∠ ACE  35.3 (cor. to 3 sig. fig.) 23 cm
∴ The angle between the line CE and the plane ABCD is
tan  
30 cm
35.3°.
  37.5 (cor. to 3 sig. fig.)
Quick Practice 6.3 (p. 6.12)
(a) Let P and Q be the mid-points of AE and DH respectively. (b) Let M be a point on AB such that NM⊥AB.
Join MP and PQ. It is noted that AE is the line of ∴ The distance between N and the line AB is MN.
intersection of the planes AME and ADHE. Consider △BCE.
∵ MP⊥AE and PQ⊥AE BC 2  BE 2  CE 2 (Pyth. theorem)
∴ The angle between the planes AME and ADHE is
∠MPQ. BC  30 2  232 cm
Consider △MPQ.
 37.8 cm (cor. to 3 sig. fig.)
QM
tan∠ MPQ  ∵ BCNM is also a rectangle.
PQ ∴ MN = BC
8 cm ∴ The distance between N and the line AB is 37.8 cm.

8 cm
∠ MPQ  45
∴ The angle between the planes AME and ADHE is 45°.

(b) Join AM and BM. It is noted that AB is the line of


intersection of the planes MAB and ABCD.
∵ BM⊥AB and BC⊥AB
∴ The angle between the planes MAB and ABCD is
∠CBM.
Consider △BCM.

207
NSS Mathematics in Action (2nd Edition) 5A Full Solutions

Quick Practice 6.6 (p. 6.20) (b) AB is the line of intersection of the planes VAB and ABCD.
The angle between the planes ADFC and BEFC is ACB. Let M be the mid-point of AB.
Consider △ABC. By the cosine formula, we have
BC 2  AC 2  AB 2
cos ACB 
2( BC )( AC )
20 2  282  14 2

2( 20)(28)
247

280
∵ △VAB and △OAB are isosceles triangles.
ACB  28.1 (cor. to 3 sig. fig.)
∴ VM  AB and OM  AB
∴ The angle between the planes ADFC and BEFC is 28.1. ∴ The angle between the planes VAB and ABCD is
VMO.
Quick Practice 6.7 (p. 6.21)
Let N be the projection of A on the plane BCFE. Then, the angle 21
OM  cm  10.5 cm
between the line AE and the plane BCFE is∠AEN and 2
AN⊥BC. Consider △VOM.
∵ AB = AC and AN⊥BC VO
∴ BN = CN (prop. of isos. △) tan VMO 
Consider △ABN. OM
AN 2  BN 2  AB 2 (Pyth. theorem) 20 cm

2 10.5 cm
 14 
AN  25 2    cm VMO  62.3 (cor. to 3 sig. fig.)
 2  ∴ The angle between the planes VAB and ABCD is
 24 cm 62.3.
Consider △ABE.
AE 2  AB 2  BE 2 (Pyth. theorem) (c) VO is the line of intersection of the planes VAO and VBO.
∵ AO  VO and BO  VO
AE  25 2  30 2 cm ∴ The angle between the planes VAO and VBO is
 1525 cm AOB.
Consider △AOB.
Consider △AEN.
OA  OB (property of rectangle)
AN
sin∠ AEN   14.5 cm
AE By the cosine formula, we have
24 cm
 OA 2  OB 2  AB 2
1525 cm cos AOB 
2(OA)(OB )
∠ AEN  37.9 (cor. to 3 sig. fig.)
14.5 2  14.5 2  20 2
∴ The angle between the line AE and the plane BCFE is 37.9°. 
2(14.5)(14.5)
Quick Practice 6.8 (p. 6.24) AOB  87.2 (cor. to 3 sig. fig.)
(a) The angle between the line VA and the plane ABCD is
VAO. ∴ The angle between the planes VAO and VBO is
Consider △ABC. 87.2.

AC  AB 2  BC 2 (Pyth. theorem) Quick Practice 6.9 (p. 6.25)


(a) Consider △ABC.
 20  21 cm
2 2
AC 2  AB 2  BC 2 (Pyth. theorem)
 29 cm
∴ AC  3  4 cm
2 2

1  5 cm
OA  AC  14.5 cm (property of rectangle)
2 Consider △ACD.
Consider △VOA. AD 2  AC 2  CD 2 (Pyth. theorem)
VO
tan VAO  AD  5  ( 11 )
2 2
cm
OA
 6 cm
20 cm
 Consider △ABD.
14.5 cm By the cosine formula, we have
VAO  54.1 (cor. to 3 sig. fig.) BD 2  AB 2  AD 2  2( AB )( AD ) cos∠ BAD
∴ The angle between the line VA and the plane ABCD is BD  32  6 2  2(3)(6) cos 60 cm
54.1.
 27 cm (or 3 3 cm)

208
6 Applications of Trigonometry in 3-dimensional Problems

(b) Consider △BCD. OH 2  OK 2  HK 2 (Pyth. theorem)


BC  CD  [4  ( 11 ) ] cm
2 2 2 2 2
(b)
OH  ( 75 ) 2  7 2 km
 27 cm 2
 26 km
BD 2  ( 27 ) 2 cm 2  27 cm 2 Consider △AOH.
∴ BC 2  CD 2  BD 2 tan 50 
AH
∴ ∠BCD = 90° (converse of Pyth. theorem) 26 km
∵ CD is the line of intersection of the planes ACD and
BCD with BC⊥CD and AC⊥CD. AH  26 tan 50 km
∴ ∠ACB is the angle between the planes ACD and BCD.  6.08 km (cor. to 3 sig. fig.)
∴ The student is correct.
(c) With the notations in the figure,
Quick Practice 6.10 (p. 6.32)
(a) Consider △LOA.
50 m
AO 
tan 50
Consider △LOB.
50 m
BO 
tan 35
AOB  270  126  144
Consider △OAB.
By the cosine formula, we have
Consider △ACB.
AC
tan  
AB 2
 AO 2
 BO 2
 2( AO )( BO ) cos A
2 2

AB 



50
tan 50 



 
BCtan
50 

35  
 50  50 
 2
 tan 50  tan 50  535
( 26 tan 
 ) km  

cos 144

 108 .1975 m 
 108 m (c or. to 3 sig. 7 km fig. )

  8.74 (cor. to 3 sig. fig.)


(b) Consider △OAB.
∴ The angle of the aeroplane descent from A to B is 8.74.
AO 2  AB 2  BO 2
cos OAB 
2( AO)( AB) Quick Practice 6.12 (p. 6.36)
2 2
(a) (i) Consider △ADF.
 50   50  AD
   108.1975   
2
tan∠ AFD 
tan 50   tan 35 
 DF
 50  8m
2 (108.1975) tan 35 
 tan 50  DF
OAB  22.8253 8
DF  m
∴ The true bearing of B from A is tan 35
(90 + 22.8253) = 113 (cor. to 3 sig. fig.).  11.4252 m
Quick Practice 6.11 (p. 6.34)  11.4 m (cor. to 3 sig. fig.)
(a) Consider △BOK. ∴ The distance between D and F is 11.4 m.
OK  10 2  52 km (Pyth. theorem)
 75 km
Consider △HOK.
∵ K is due east of H.
∴ KHO  90

HK
sin HOK 
OK (ii)
7 km
 With the notations in the figure,
75 km ∠ FDR  ∠ PDQ  22 (vert. opp.∠s)
HOK  53.9 (cor. to 3 sig. fig.) ∠BDF = 90° + 22° = 112°
∴ The compass bearing of B from O is N53.9E. ∠ BDF  ∠ CDF  180 (adj. s on st. line)

112   ∠ CDF  180
∠ CDF  68
Consider △ABD.
∵ D is the projection of A on the horizontal ground.

209
NSS Mathematics in Action (2nd Edition) 5A Full Solutions

∴ ∠ADB = 90° 8 sin 68  8 sin 68 


AD 2  BD 2  AB 2 (Pyth. theorem)  2 
tan   tan 35 
BD  112  8 2 m ∴
tan 35
tan  
 7.5498 m 2
CD  (13  7.5498) m   19.3 (cor. to 3 sig. fig.)
 5.4502 m
Area of the shadow FBC Quick Practice 6.13 (p. 6.38)
 10 
 area of △ BDF  area of △ CDF (a) AP  BP    cm  5 cm
 2 
1
  BD  DF  sin∠ BDF Consider △ABP in Figure (b).
2 By the cosine formula, we have
1 AB 2  AP 2  BP 2  2( AP )( BP) cos∠ APB
  CD  DF  sin∠ CDF
2
1  AB  5 2  5 2  2(5)(5) cos 50 cm
  7.5498  11 .4252  sin 112  
  2  m2  4.2262 cm
 1 
  5.4502  11 .4252  sin 68   4.23 cm (cor. to 3 sig. fig.)
 2  ∴ The distance between A and B is 4.23 cm.
 68.9 m 2 (cor. to 3 sig. fig.)
Alternative Solution (b) Consider △APQ.
With the notations in the figure, AQ 2  AP 2  PQ 2 (Pyth. theorem)
∠ FDR  ∠ PDQ  22 (vert. opp.∠s)
AQ  5  12 cm
2 2
FDE  90  22
 13 cm
 68
Similarly, BQ  13 cm
 FD sin FDC Consider △ABQ in Figure (b).
8 By the cosine formula, we have
Height of △FBC   sin 68 m
tan 35 AQ 2  BQ 2  AB 2
cos∠ AQB 
8 sin 68 2( AQ)( BQ)
 m
tan 35 132  132  4.2262 2
Area of the shadow FBC 
2(13)(13)
1
  BC  height of △ FBC ∠ AQB  18.7 (cor. to 3 sig. fig.)
2
1 8 sin 68 2 (c) Let M be a point on AB such that QM⊥AB. Join QM and
  13  m
2 tan 35 PM.
∵ AQ = BQ and QM⊥AB
 68.9 m 2 (cor. to 3 sig. fig.)
∴ AM = BM (prop. of isos. △)
∵ AP = BP and AM = BM
(b) When the angle of elevation of the sunshine has changed to ∴ PM⊥AB (prop. of isos. △)
8 ∵ AB is the line of intersection of the planes QAB and the
, DF  m. horizontal table with QM⊥AB and PM⊥AB.
tan  ∴ The angle between the plane QAB and the horizontal
∵ The area of the shadow of the plate on the ground is table is∠PMQ.
two times that obtained in (a). Consider △AMP.

50
 1    ∠ APM   25
2
8
  BD   m   sin ∠ BDF 
2  tan 35 
2   
 1  8  
  CD   m   sin ∠ CDF 

 2  tan 35  


1 
 BD  
8 
m   sin ∠ BD F MP
cos∠ APM 
2  tan  
1  8 
  CD   m   sin ∠ CDF
 tan 
AP
2 
2 1

tan 35 tan 
tan 35

MP
tan 
2

cos 25 
  19.3 (cor. to 3 sig. fig.)

Alternative Solution
When the angle of elevation of the sunshine has changed to
5 cm
8 MP  5 cos 25 cm
 DF  m Consider △PMQ.
tan 
PQ
8 sin 68 tan∠ PMQ 
∴ New height of the shadow  m MP
tan  12 cm
∵ The area of the shadow of the plate on the ground is 
two times that obtained in (a). 5cos25 cm
i.e. The height of the shadow of the plate on the ground is ∠ PMQ  69.3 (cor. to 3 sig. fig.)
two times its original height.
∴ The angle between the plane QAB and the horizontal
table is 69.3°.

210
6 Applications of Trigonometry in 3-dimensional Problems

(b) Consider △AMB.


Quick Practice 6.14 (p. 6.40) 80
(a) With the notation in the figure, BAM 
2
 40
AM
cos BAM 
AB
AM
cos 40 
8 cm
Let M be the mid-point on BD. Join CM and extend AM to
N. AM  8 cos 40 cm
∵ BD is the line of intersection of the plane CBD and the DN  AM
horizontal table with CM⊥BD and AN⊥BD.  6.13 cm (cor. to 3 sig. fig.)
∴ The angle between the plane CBD and the horizontal
table is∠CMN.
(c) ∵ BC is the line of intersection of the planes DBC and
Consider △ABM.
BCFE with DM⊥BC and MN⊥BC.
AM 2  BM 2  AB 2 (Pyth. theorem) ∴ The angle between the planes DBC and BCFE is
2 DMN.
8 Consider △DMN.
AM  8 2    cm
2 DN
tan DMN 
 48 cm MN
Similarly, CM  48 cm 8 cos 40 cm

Consider △ABC. 14 cm
AC 2  AB 2  BC 2 (Pyth. theorem) DMN  23.6 (cor. to 3 sig. fig.)
AC  8 2  8 2 cm ∴ The angle between the planes DBC and BCFE is
23.6.
 128 cm
Consider △ACM. 2. (a) Let N be a point on VB such that AN⊥VB.
By the cosine formula, we have ∵ VA = AB and AN⊥VB
AM 2  CM 2  AC 2 ∴ VN = BN (prop. of isos. △)
cos∠ AMC  Consider △ABN.
2( AM )(CM )
AN 2  BN 2  AB 2 (Pyth. theorem)
( 48 ) 2  ( 48 ) 2  ( 128 ) 2
  12 
2

2( 48 )( 48 ) AN  12 2    cm
 2 
∠ AMC  109.4712
 108 cm
∠ CMN  ∠ AMC  180 (adj. s on st. line)  10.4 cm (cor. to 3 sig. fig.)
∴ The perpendicular distance between A and VB is
∠ CMN  180  109.4712 10.4 cm.
 70.5288
 70.5 (cor. to 3 sig. fig.) (b) Join CN.
∵ VC = BC and VN = BN
∴ The angle between the plane CBD and the horizontal ∴ CN⊥VB (prop. of isos. △)
table is 70.5°. ∵ VB is the line of intersection of the planes VAB
and VBC with AN⊥VB and CN⊥VB.
(b) Let P be the projection of C on the horizontal table. ∴ The angle between the planes VAB and VBC is
Consider △CMP. ∠ANC.
CP Also, CN  AN  108 cm
sin∠ CMP 
CM Consider △ABC.
CP  48 sin 70.5288 cm AC 2  AB 2  BC 2 (Pyth. theorem)
 6.53 cm (cor. to 3 sig. fig.) AC  12 2  12 2 cm
∴ The shortest distacne between the point C and the
 288 cm
horizontal table is 6.53 cm.
Consider △ACN.
By the cosine formula, we have
Further Practice
AN 2  CN 2  AC 2
cos∠ ANC 
Further Practice (p. 6.26) 2( AN )(CN )
1. (a)
( 108 ) 2  ( 108 )   ( 288 ) 2

CD  14 2  82 cm (Pyth. theorem) 2( 108 )( 108 )
 16.1 cm (cor. to 3 sig. fig.) ∠ ANC  109 (cor. to 3 sig. fig.)

211
NSS Mathematics in Action (2nd Edition) 5A Full Solutions

∴ The angle between the planes VAB and VBC is BC


109°. cos∠ CBD 
BD
Further Practice (p. 6.41) 5m
1. (a) Consider △DCQ. cos 30 
BD
5m
tan 20  5
CQ BD  m
cos 30
5
CQ  m
tan 20
Area of BPQC
 5  2
 12  m
 tan 20 
 165 m 2 (cor. to 3 sig. fig.)

(b)

Consider △ABP.
5
BP  CQ  m
tan 20
Let F be a point on BC such that BC  FP.
Consider △BFP.

RBP  BPF  15 (alt. s, BR // FP)


FP
cos BPF 
BP
FP
cos 15 
5
m
tan 20
5 cos15
FP  m
tan 20
Area of BPQC
5 cos15
 12  m2
tan 20
 159 m 2 (cor. to 3 sig. fig.)

2. (a) Consider △BCD.


CD
tan∠ CBD 
BC
CD
tan 30 
5m
CD  5 tan 30 m
FE  CD
 5 tan 30 m
 2.89 m (cor. to 3 sig. fig.)

212
6 Applications of Trigonometry in 3-dimensional Problems

Consider △FDB.
FD 2  BF 2  BD 2 (Pyth. theorem)
2
 5 
FD  20 2    m Consider △BCE.
 cos 30  By the cosine formula, we have
 19.1485 m BC 2  BE 2  EC 2
cos EBC 
 19.1 m (cor. to 3 sig. fig.) 2( BC )( BE )
( 28 ) 2  ( 29 ) 2  ( 41 ) 2
(b) The angle of inclination of the plane ABEF is∠EBC. 
Consider △EBC. 2( 28 )( 29 )
EC EBC  73.6953
tan∠ EBC 
BC  73.7 (cor. to 3 sig. fig.)
FD

BC (c) Let G be a point on BC such that EG  BC and
19.1485 m FG  BC.
 The angle between the planes BCE and BCF is EGF.
5m
∠ EBC  75.4 (cor. to 3 sig. fig.)
∴ The angle of inclination of the plane ABEF is
75.4°.

(c) Let H be the projection of G on the plane ABCD.


∴ H is the mid-point of CD.
CD
CH  Consider △BGE.
∴ 2
5 tan 30 EG
 m sin EBG 
2 BE
Consider △BCH. EG
sin 73.6953 
CH 29 cm
tan∠ CBH 
BC EG  5.1686 cm
5 tan 30 Consider △EGF.
m EF
 2 sin EGF 
5m EG
∠ CBH  16.1 (cor. to 3 sig. fig.) 5 cm

∴ The compass bearing of G from B is N16.1°W. 5.1686 cm
EGF  75.3 (cor. to 3 sig. fig.)
3. (a) ED  2 AE  2  2 cm  4 cm ∴ The angle between the planes BCE and BCF is
Consider △BFC. 75.3.
By the cosine formula, we have
Exercise
BC 2  FB 2  FC 2  2( FB )( FC ) cos BFC
BC  2 2  4 2  2(2)( 4) cos 120 cm Exercise 6A (p. 6.16)
Level 1
 28 cm 1. (a) Join AH and CH.
 5.29 cm (cor. to 3 sig. fig.) ∵ AH and CH intersect at H.
∴ The angle between the lines AH and CH is AHC.

BE 2  AE 2  AB 2 (b) Join AG and DG.


(b) (Pyth. theorem) ∵ AG and DG intersect at G.
BE  2  5 cm
2 2
∴ The angle between the lines AG and DG is AGD.
 29 cm
(c) Join AH.
EC  ED 2  DC 2
2
∵ DH is the projection of AH on the plane CGHD.
(Pyth. theorem)
EC  4 2  52 cm ∴ The angle between the line AH and the plane
CGHD is AHD.
 41 cm
(d) ∵ EH is the projection of AH on the plane EFGH.
∴ The angle between the line AH and the plane
EFGH is AHE.

(e) Join BE and CH.


BC is the line of intersection of the planes BCGF and
213
NSS Mathematics in Action (2nd Edition) 5A Full Solutions

BCHE. ∴ The angle between the planes BCGF and BCHE is


∵ FB  BC and EB  BC (or GC  BC and EBF or HCG.
HC  BC)
(f) Join AC and EG.
AE is the line of intersection of the planes ACGE and
ABFE.
∵ EG  AE and EF  AE (or AC  AE and
AB  AE)
∴ The angle between the planes ACGE and ABFE
is GEF or CAB.

2. (a) Join AH.


∵ The projection of H on the plane ABCD is D.
∴ AD is the projection of AH on the plane ABCD.

(b) (i) ∵ AH and AD intersect at A.


∴ The angle between the lines AH and AD is
∠DAH.
Consider △ADH.

DH
tan∠ DAH 
AD
2 cm

4 cm
∠ DAH  26.6 (cor. to 3 sig. fig.)
∴ The angle between the lines AH and AD is
26.6°.
(ii) ∵ AD is the projection of AH on the plane
ABCD.
∴ The angle between the line AH and the plane
ABCD is∠DAH, i.e. 26.6°.

3. (a) In △ABC,
AC  4 2  32 cm (Pyth. theorem)
 5 cm
In △ACG,
AG 2  AC 2  GC 2 (Pyth. theorem)
AG  5 2
2 2
cm
 29 cm

(b) (i) The angle between the lines AG and GH is∠AGH.

GH
cos∠ AGH 
AG
4 cm

29 cm
∠ AGH  42.0 (cor. to 3 sig. fig.)
∴ The angle between the lines AG and GH is
42.0°.
(ii) The angle between the line AG and the plane
EFGH is∠AGE.
EG  AC  5 cm
AE
tan∠ AGE 
EG
2 cm

5 cm
∠ AGE  21.8 (cor. to 3 sig. fig.)
∴ The angle between the line AG and the plane
EFGH is 21.8°.

214
6 Applications of Trigonometry in 3-dimensional Problems

ADHE is 45.7°.
4. (a) BD  52  5 2 cm (Pyth. theorem)
 50 cm

2
BH  BD 2  DH 2 (Pyth. theorem)
6. (a) ∵ The projection of B on the plane CGHD is C.
BH  ( 50 ) 2  5 2 cm
∴ The projection of HB on the plane CGHD is HC.
 75 cm (or 5 3 cm) HC  7 2  4 2 cm (Pyth. theorem)
 65 cm
(b) (i) The angle between the lines BH and BA is ABH. ∴ The length of the projection of HB on the plane
CGHD is 65 cm .

AB (b) (i) The angle between the lines HB and DB is HBD.


cos ABH  In △ABD,
BH
BD 2  AB 2  AD 2 (Pyth. theorem)
5

75 BD  7 2  52 cm

ABH  54.7 (cor. to 3 sig. fig.)  74 cm


∴ The angle between the lines BH and BA is In △HBD,
54.7.
(ii) The angle between the line BH and the plane HD
CGHD is BHC. tan HBD 
BD
4
BC 
sin BHC  74
BH
5 HBD  24.9 (cor. to 3 sig. fig.)
 ∴ The angle between the lines HB and DB is
75 24.9°.
BHC  35.3 (cor. to 3 sig. fig.) (ii) The angle between the line HB and the plane
∴ The angle between the line BH and the plane CGHD is BHC.
In △BHC,
CGHD is 35.3.

5. (a) The angle between the lines AG and GC is AGC. BC


In △ABC, tan BHC 
HC
AC 2  AB 2  BC 2 (Pyth. theorem) 5

AC  8  6 cm
2 2
65
 10 cm BHC  31.8 (cor. to 3 sig. fig.)
In △AGC, ∴ The angle between the line HB and the plane
AC CGHD is 31.8°.
tan AGC 
CG
7. (a) BE (or AF)
10

5 (b) Consider △FAD.
AGC  63.4 (cor. to 3 sig. fig.) FD
tan∠ FAD 
∴ The angle between the lines AG and GC is 63.4°. AD
4
(b) The angle between the line AG and the plane ADHE is 
GAH. 5
In △HAD, ∠ FAD  38.7 (cor. to 3 sig. fig.)
AH 2  AD 2  DH 2 (Pyth. theorem) ∴ The inclination of the line of greatest slope of the
plane ABEF is 38.7°.
AH  6 2  52 cm
 61 cm 8. (a) Consider △FBE.
In △GAH, 6 cm
cos 48 
HG FB
tan GAH 
AH 6
FB  cm
8 cos 48
  8.97 cm (cor. to 3 sig. fig.)
61
GAH  45.7 (cor. to 3 sig. fig.)
(b) The angle between the line FB and the plane ABCD
∴ The angle between the line AG and the plane
215
NSS Mathematics in Action (2nd Edition) 5A Full Solutions

is∠FBD. Consider △FDB.


Consider △EBC. FD
EC sin∠ FBD 
sin 35  FB
6 cm 6 sin 35
EC  6 sin 35 cm 
6
FD  EC
cos 48
 6 sin 35 cm  sin 35 cos 48
∠ FBD  22.6 (cor. to 3 sig. fig.)
∴ The angle between the line FB and the plane
ABCD is 22.6°.

9. (a) Consider △BDF.


700 m
tan 20 
FB
700
FB  m
tan 20
 1920 m (cor. to 3 sig. fig.)

(b) ∵ BC is the line of greatest slope of the hillside


ABCD.
∴ ∠CBE is the inclination of the line of greatest
slope of the hillside ABCD.
Consider △BDF.
700 m
sin 20 
BD
700
BD  m
sin 20
Consider △BCD.
BC
cos 55 
700
m
sin 20
700 cos 55
BC  m
sin 20
Consider △BCE.
700 m
sin∠ CBE 
700cos55
m
sin20
∠ CBE  36.6 (cor. to 3 sig. fig.)
∴ The inclination of the line of greatest slope of the
hillside ABCD is 36.6°.

10. (a) AC  10 2  10 2 cm (Pyth. theorem)


 200 cm
AG 2  AC 2  CG 2 (Pyth. theorem)

AG  ( 200 ) 2  10 2 cm
 300 cm
300
AM  cm
2
Similarly, BM 
300
cm
2
Let N be the mid-point of AB.
∵ AM  BM and AN  BN
∴ MN⊥AB (prop. of isos. △)
∴ The distance between the point M and the line AB
is MN.

216
6 Applications of Trigonometry in 3-dimensional Problems

AN 2  MN 2  AM 2 (Pyth. theorem) AC  10 2  10 2 cm (Pyth. theorem)


2
  2  200 cm
MN   300    10  cm
 2   2  200
  CM  cm
 50 cm
2
Consider △CGM.
 7.07 cm (cor. to 3 sig. fig.) 10
∴ The distance between the point M and the line AB tan∠ CMG 
is 7.07 cm. 200
2
(b) Let P be the projection of M on the plane ABCD.
The distance between the point M and the plane ABCD
∠ CMG  54.7 (cor. to 3 sig. fig.)
is MP. ∴ The angle between the planes BDG and ABCD is 54.7°.
∠APM = 90°
13. (a) The angle between the planes ABGH and ABCD is
1 200 HAD.
AP  AC  cm
2 2 Consider △HAD.
AP 2  MP 2  AM 2 (Pyth. theorem) 9
tan HAD 
 300 
2
 
2 9
MP      200  cm HAD  45
 2   2 
    ∴ The angle between the planes ABGH and ABCD is
 5 cm 45.
∴ The distance between the point M and the plane
ABCD is 5 cm. (b) The angle between the planes ABGH and PQGH is
∠BGQ.
Level 2 BG  9 2  9 2 cm (Pyth. theorem)
11. (a) The angle between the planes AFGD and ABCD is
∠FAB.  162 cm
Consider △FAB. ∵ AP : PD  BQ : QC  2 : 1
3 2
tan∠ FAB  ∴ BQ  9  cm  6 cm
8 3
∠ FAB  20.5560 1
 20.6 (cor. to 3 sig. fig.) and QC  9  cm  3 cm
3
∴ The angle between the planes AFGD and ABCD is
20.6°. GQ  9 2  3 2 cm (Pyth. theorem)
 90 cm
(b) Let RS be the line of intersection of the planes AFGD
Consider △BGQ.
and EPQH.
By the cosine formula,
∵ ER⊥RS and AR⊥RS
∴ ∠ERA is the angle between the planes AFGD and BG 2  GQ 2  BQ 2
EPQH.
cos∠ BGQ 
2( BG )(GQ)
Consider △EAP.
3 ( 162 ) 2  ( 90 ) 2  6 2
tan∠ EPA  
4 2( 162 )( 90 )
∠ EPA  36.8699 ∠ BGQ  26.6 (cor. to 3 sig. fig.)
Consider △APR. ∴ The angle between the planes ABGH and PQGH is
26.6°.
∠ ERA  ∠ EPA ∠ FAB (ext.∠ of △)
14. Let M be the mid-point of PQ.
 36.8699  20.5560 ∵ GP  GQ and PM  QM
 57.4 (cor. to 3 sig. fig.) ∴ GM⊥PQ (prop. of isos. △)
∴ The angle between the planes AFGD and EPQH is ∵ CP  CQ and PM  QM
57.4°. ∴ CM⊥PQ (prop. of isos. △)
∵ PQ is the line of intersection between the planes GPQ
12. Join BD, BG and DG. and ABCD with GM⊥PQ and CM⊥PQ.
Let M be the mid-point of BD. ∴ The angle between the planes GPQ and ABCD is
∵ CB = CD and BM = DM ∠GMC.
∴ CM⊥BD (prop. of isos. △)
∵ BG = DG and BM = DM
∴ GM⊥BD (prop. of isos. △) 1  12   12 
∵ BD is the line of intersection of the planes BDG and Area of △ CPQ   cm    cm   18 cm 2
2  2   2 
ABCD with CM⊥BD and GM⊥BD.
∴ The angle between the planes BDG and ABCD is
∠CMG.

217
NSS Mathematics in Action (2nd Edition) 5A Full Solutions

PQ 2  CP 2  CQ 2 (Pyth. theorem) BF
tan BGF 
 12 
2
 12 
2 FG
PQ      cm 14
 2   2  
18
 72 cm
BGF  37.9 (cor. to 3 sig. fig.)
∵ Area of △CPQ = 18 cm2
∴ The angle between the planes KGH and EFGH is
1 37.9°.
 72 cm  CM  18 cm 2
∴ 2
36 9
2
CM  cm PE     6 cm
2
(Pyth. theorem)
72 16. (a) 2
Consider △CGM.  7.5 cm
GC
tan∠ GMC  9
2
CM PC     7 2 cm (Pyth.theorem)
12 2

36  69.25 cm
72  8.32 cm (cor. to 3 sig. fig.)

∠ GMC  70.5 (cor. to 3 sig. fig.) AC  9 2  7 2 cm (Pyth. theorem)


∴ The angle between the planes GPQ and ABCD is 70.5°.  130 cm
15. (a) Consider △EFG. EC 2
 AC 2  AE 2 (Pyth. theorem)
EG 2  EF 2  FG 2 (Pyth. theorem) EC  ( 130 )  6 2 2
cm
EG  10  18 cm
2 2
 166 cm
 424 cm  12.9 cm (cor. to 3 sig. fig.)
Consider △AEG.
AG 2  AE 2  EG 2 (Pyth. theorem) (b) The angle between the lines PE and EC is∠CEP.
Consider △CEP.
AG  14 2  ( 424 ) 2 cm By the cosine formula,
 620 cm PE 2  EC 2  PC 2
cos∠ CEP 
∵ ABGH is a rectangle. 2( PE )( EC )
1
KG  AG 7.52  ( 166 ) 2  ( 69.25 ) 2
∴ 2 (property of rectangle) 
2(7.5)( 166 )
620
 cm ∠ CEP  37.7 (cor. to 3 sig. fig.)
2 ∴ The angle between the lines PE and EC is 37.7°.

17. (a) Let BC = x cm.


620
KH  KG  cm (property of rectangle) Consider △BCN.
2 CN
Consider △KGH. tan 55 
By the cosine formula, x cm
CN  x tan 55 cm
KG 2  KH 2  GH 2 Consider △ADN.
cos GKH  DN
2( KG )( KH ) tan 25 
2 2 x cm
 620    620
     10 2 DN  x tan 25 cm
 2    2 CN + DN = 20 cm
    ∵
 620  620  x tan 55  x tan 25  20
2  
x(tan 55  tan 25)  20
 2  2 
  
∴ 20
GKH  47.4 (cor. to 3 sig. fig.) x
tan 55  tan 25
(b) ∵ The angle between the planes KGH and EFGH   10.5571
the angle between the planes ABGH and EFGH  10.6 (cor. to 3 sig. fig.)
∴ The angle between the planes KGH and EFGH is BC  10.6 cm

BGF.
Consider △BFG.
(b) Let M be the projection of N on the horizontal plane.
The shortest distance between the point N and the
horizontal plane is MN.

218
6 Applications of Trigonometry in 3-dimensional Problems

Consider △BCE.
CE
sin∠ CBE 
BC
CE  10.5571sin 30 cm
 5.28 cm (cor. to 3 sig. fig.)
∵ CDFE is a rectangle.
∴ MN = CE = 5.28 cm
∴ The shortest distance between the point N and the
horizontal plane is 5.28 cm.
(b)
Exercise 6B (p. 6.26)
Level 1 Let M be the mid-point of QR.
1. The angle between the planes ACFD and BCFE is ACB. QR
(a) In △ABC, by the sine formula, we have QM 
2
AB AC i.e.
 8
sin ACB sin ABC  cm
2
11 cm 9 cm
  4 cm
sin ACB sin 35 ∵ △AQR and △PQR are isosceles triangles.
11 sin 35 ∴ AM  QR and PM  QR
sin ACB  ∴ The angle between the planes AQR and PQR is
9
AMP.
ACB  44.5 (cor. to 3 sig. fig.) Consider △PQM.
∴ The angle between the planes ACFD and BCFE is PM 2  PQ 2  QM 2 (Pyth. theorem)
44.5°.
PM  52  4 2 cm
(b) In △ABC, by the cosine formula, we have
 3 cm
CA2  BC 2  AB 2 Consider △APM.
cos ACB 
2(CA)( BC ) AP
tanAMP 
10  12  8
2 2 2
PM

2(10)(12) 10 cm

3 3 cm
 AMP  73.3 (cor. to 3 sig. fig.)
4
ACB  41.4 (cor. to 3 sig. fig.) ∴ The angle between the planes AQR and PQR is
73.3°.
∴ The angle between the planes ACFD and BCFE is
41.4°. 3. (a) Let P be the projection of D on the plane BCFE.
The shortest distance between D and the plane BCFE is
2. (a) The angle between the planes ABQP and ACRP is DP.
BAC. Consider △DFP.
Consider △ABC. DP
By the cosine formula, we have sin 70 
8 cm
AB  AC  BC
2 2 2
cos BAC  DP  8 sin 70 cm
2( AB )( AC )
 7.52 cm (cor. to 3 sig. fig.)
52  52  82 ∴ The shortest distance between D and the plane

2(5)(5) BCFE is 7.52 cm.
7 (b) The angle between the line CD and the plane BCFE is

25 ∠DCP.
BAC  106 (cor. to 3 sig. fig.) CD  15 2  8 2 cm (Pyth. theorem)
∴ The angle between the planes ABQP and ACRP is  17 cm
106°. Consider △CDP.
DP
sin∠ DCP 
CD
8 sin 70 cm

17 cm
∠ DCP  26.2 (cor. to 3 sig. fig.)
∴ The angle between the line CD and the plane
BCFE is 26.2°.

4. (a) Let M be the mid-point of QR.

219
NSS Mathematics in Action (2nd Edition) 5A Full Solutions

∵ AQ = AR and QM = RM (b) Volume of the prism


∴ AM⊥QR (prop. of isos. △)  area of △ PQR  AP
∵ PQ = PR and QM = RM
∴ PM⊥QR (prop. of isos. △) 1 
   14  24  24 tan 65  cm 3
∵ QR is the line of intersection of the planes AQR 2 
and PQR with AM⊥QR and PM⊥QR.
∴ ∠AMP is the angle between the planes AQR and  8650 cm 3 (cor. to 3 sig. fig.)
PQR, i.e.∠AMP = 65°.
Consider △PQM. 5. (a) The angle between the line VA and the plane ABCD is
PM 2  QM 2  PQ 2 (Pyth. theorem) VAN.
2 AC  12 2  92 cm (Pyth. theorem)
 14 
PM  25 2    cm  15 cm
 2  ∴
 24 cm 1
Consider △AMP. AN  AC  7.5 cm (property of rectangle)
2
AP Consider △VAN.
tan 65 
24 cm 18 cm
AP  24 tan 65 cm tan VAN 
7.5 cm
 51.5 cm (cor. to 3 sig. fig.) VAN  67.4 (cor. to 3 sig. fig.)
∴ The angle between the line VA and the plane
ABCD is 67.4°.

(b)

AB is the line of intersection of the planes VAB and


ABCD.
Let M be the mid-point of AB.
∵ △VAB and △NAB are isosceles triangles.
∴ VM  AB and NM  AB
∴ The angle between the planes VAB and ABCD is
VMN.
9
MN  cm
2
 4.5 cm
Consider △VMN.
18 cm
tan VMN 
4.5 cm
VMN  76.0 (cor. to 3 sig. fig.)
∴ The angle between the planes VAB and ABCD is
76.0°.

6. (a) The angle between the line VB and the plane ABC is
∠VBA.
Consider △VAB.
8 cm
tan∠ VBA 
10 cm
∠ VBA  38.7 (cor. to 3 sig. fig.)
∴ The angle between the line VB and the plane ABC
is 38.7°.

(b) The angle between the lines VB and VC is∠BVC.


VB  8 2  10 2 cm (Pyth. theorem)
 164 cm
Similarly, VC  164 cm

220
6 Applications of Trigonometry in 3-dimensional Problems

Consider △VCM.
BC  10 2  10 2 cm (Pyth. theorem)
VM
 200 cm tan VCM 
CM
Consider △VBC.
By the cosine formula, we have 10 cm

VB 2  VC 2  BC 2 45.25 cm
cos∠ BVC 
2(VB)(VC ) VCM  56.1 (cor. to 3 sig. fig.)
∴ The angle between the line VC and the plane
( 164 ) 2  ( 164 ) 2  ( 200 ) 2
 ABCD is 56.1°.
2( 164 )( 164 )
(b) Let N be the mid-point of CD.
∠ BVC  67.0 (cor. to 3 sig. fig.)
∴ The angle between the lines VB and VC is 67.0°.

7. (a) Join AC and BD. Let AC and BD intersect at P.


The shortest distance between V and the plane ABCD is
VP.
Consider △ABC.
AC  10 2  10 2 cm (Pyth. theorem)
 200 cm
200 ∵ DC is the line of intersection of the planes VDC
AP  cm and ABCD with VN  DC and MN  DC.
2 ∴ The angle between the planes VDC and ABCD is
Consider △VAP. VNM.
AP 2  VP 2  VA 2 (Pyth. theorem) Consider △VNM.
2 VM
 200  tan VNM 
VP  10 2    cm MN
 2 
  10 cm
 50 cm

5 cm
 7.07 cm (cor. to 3 sig. fig.) VNM  63.4 (cor. to 3 sig. fig.)
∴ The shortest distance between V and the plane ∴ The angle between the planes VDC and ABCD is
ABCD is 7.07 cm. 63.4°.
(b) The angle between the lines VM and VN is∠MVN. 9. (a) The angle between the covers ABCD and BCEF is
DCE.
2 Consider △DCE.
 10 
VM  10 2    cm (Pyth. theorem) CD 2  CE 2  DE 2
 2  cos DCE 
2(CD )(CE )
 75 cm
252  252  82
Similarly, VN  75 cm 
2(25)(25)
MN  AB  10 cm
Consider △VMN. DCE  18.4138
By the cosine formula, we have  18.4 (cor. to 3 sig. fig.)
VM 2  VN 2  MN 2 ∴ The angle between the covers ABCD and BCEF is
cos∠ MVN 
2(VM )(VN ) 18.4°.

( 75 ) 2  ( 75 ) 2  10 2

2( 75 )( 75 )
∠ MVN  70.5 (cor. to 3 sig. fig.)
∴ The angle between the lines VM and VN is 70.5°.

8. (a) The angle between the line VC and the plane ABCD is
VCM. (b)
1 The angle between the line BD and the plane BCEF is
BM  AB  4.5 cm
2 DBG.
Consider △BCM. Consider △BCD.
CM 2  BM 2  BC 2 (Pyth. theorem) BD 2  BC 2  CD 2 (Pyth. theorem)

CM  4.5 2  5 2 cm BD  202  252 cm


 45.25 cm  1025 cm

221
NSS Mathematics in Action (2nd Edition) 5A Full Solutions

Consider △DCG. Consider △VBK.


DG VK
sin DCG  tan VBK 
CD BK
DG  25 sin 18.4138 cm VK
tan 15 
 7.8969 cm 12.5 m
Consider △DBG. VK  12.5 tan 15 m
DG Height of the cottage
sin DBG 
BD
7.8969 cm  (12.5 tan 15  7) m
  10.3 m (cor. to 3 sig. fig.)
1025 cm
DBG  14.3 (cor. to 3 sig. fig.)
∴ The angle between the line BD and the plane
BCEF is 14.3°.

10. (a) Join AM.


∵ AB = AC and BM = CM
∴ ∠BAM =∠CAM and AM⊥BC
(prop. of isos. △)
40
∴ ∠BAM   20
2 (b)
Consider △ABM.
Let M and N be the mid-points of AD and FE
AM respectively.
cos∠ BAM 
AB ∵ AD is the line of intersection of the planes VAD
AM and ADEF with VM  AD and MN  AD.
cos 20  ∴ The angle between the planes VAD and ADEF is
10 cm
VMN.
AM  10 cos 20 cm
15
 9.40 cm (cor. to 3 sig. fig.) KM  m
2
 7.5 m
(b) ∵ AB = AC and AD is perpendicular to the plane
Consider △VMK.
DBC.
∴ The length of the projection of AB on the plane VK
tan VMK 
DBC = the length of the projection of AC on the KM
plane DBC,
i.e. DB = DC. 12.5 tan 15 m

∵ DB = DC and BM = CM 7.5 m
∴ DM⊥BC (prop. of isos. △)
∵ BC is the line of intersection of the planes ABC
VMK  24.065
and DBC with AM⊥BC and DM⊥BC. VMN  VMK  KMN
∴ The angle between the planes ABC and DBC is  24.065  90
∠AMD.
Consider △ADM.  114  (cor. to 3 sig. fig.)
AD ∴ The angle between the planes VAD and ADEF is
sin∠ AMD  114°.
AM
5 cm 12. (a) In △VBA,

10 cos 20 cm
∠ AMD  32.1 (cor. to 3 sig. fig.) AB  15 2  ( 56 ) 2 cm (Pyth. theorem)
∴ The angle between the planes ABC and DBC is  13 cm
32.1°. In △VCA,

11. (a) Consider △BCD.


BD 2  BC 2  CD 2 (Pyth. theorem) AC  92  ( 56 ) 2 cm (Pyth. theorem)
 5 cm
BD  20 2  15 2 m
 25 m (b) In △VBC,
1 BC 2  VC 2  (12 2  9 2 ) cm 2
BK  BD (property of rectangle)
2  225 cm 2
 12.5 m VB  15 cm 2  225 cm 2
2 2

∴ BC 2  VC 2  VB 2
∴ ∠VCB = 90° (converse of Pyth. theorem)

222
6 Applications of Trigonometry in 3-dimensional Problems

In △ABC, AM 2
 AB 2  BM 2 (Pyth. theorem)
AC  BC  (5  12 ) cm
2 2 2 2 2
AM  10  5
2 2
cm
 169 cm 2  75 cm (or 5 3 cm)
AB 2  13 2 cm 2  169 cm 2
∴ AC 2  BC 2  AB 2
∴ ∠ACB = 90° (converse of Pyth. theorem)
∵ BC is the line of intersection of the planes VBC
and ABC with VC⊥BC and AC⊥BC.
∴ ∠VCA is the angle between the planes VBC and
ABC.
∴ The student is correct.

Level 2
13. (a) Let M be the mid-point of AB.
∵ VA = VB and AM = BM
∴ VM⊥AB (prop. of isos. △)
∵ NA = NB and AM = BM
∴ NM⊥AB
∵ AB is the line of intersection of the planes VAB
and ABCD with VM⊥AB and NM⊥AB.
∴ The angle between the planes VAB and ABCD is
∠VMN.
4 cm
MN   2 cm
2
Consider △VMN.
VN
tan 72 
2 cm
VN  2 tan 72 cm
 6.16 cm (cor. to 3 sig. fig.)

(b) The angle between the lines VA and VC is∠AVC.


In △ABC,
AC  4 2  4 2 cm (Pyth. theorem)
 32 cm
32
AN  CN  cm
2
In △VAN,
VA 2  AN 2  VN 2 (Pyth. theorem)
2
 32 
VA     (2 tan 72) 2 cm
 2 
 
 6.7741 cm
Similarly, VC  6.7741 cm
Consider △AVC.
By the cosine formula, we have
VA 2  VC 2  AC 2
cos∠ AVC 
2(VA )(VC )
6.77412  6.77412  ( 32 ) 2

2(6.7741)(6.7741)
∠ AVC  49.4 (cor. to 3 sig. fig.)
∴ The angle between the lines VA and VC is 49.4°.

10
14. (a) BM  cm  5 cm
2
Consider △ABM.

223
NSS Mathematics in Action (2nd Edition) 5A Full Solutions

(b) Let N be the mid-point of AF. Consider △VBM.


VM  VB 2  MB 2 (Pyth. theorem)
2 2
 8   4 
     cm
 sin 40    tan 40  
 11 .4967 cm
 11 .5 cm (cor. to 3 sig. fig.)

(b) ∵ AB is the line of intersection of the planes VAB


and CAB with VM⊥AB and CM⊥AB.
∴ The angle between the planes VAB and CAB is
∠VMC.
Consider △VMC.
10
NM  cm  5 cm VC
2 sin∠ VMC 
Consider △ANM. VM
AN 2  AM 2
 NM 2 (Pyth. theorem) 8 cm

AN  ( 75 ) 2  5 2 cm
11 .4967 cm
∠ VMC  44.1 (cor. to 3 sig. fig.)
 50 cm
∴ The angle between the planes VAB and CAB is
 5 2 cm 44.1°.
AF  2  5 2 cm
16. (a) Consider △ABH.
 10 2 cm (or 200 cm )
AB
cos ABH 
(c) BC is the line of intersection of the planes ABC and
BH
BFC. AB
cos 30 
∵ AM  BC and FM  BC 40 cm
∴ The angle between the planes ABC and BFC is
AMF.
AB  40 cos 30 cm
AH
FM  AM  75 cm sin∠ ABH 
Consider △AFM. BH
By the cosine formula, we have AH  40 sin 30 cm
AM 2  FM 2  AF 2  20 cm
cos AMF 
2( AM )( FM ) Consider △ACH.
AH
( 75 )  ( 75 )  (10 2 )
2 2 2
tan ACH 
 AC
2( 75 )( 75 )
20 cm
AMF  109 (cor. to 3 sig. fig.) tan 60 
AC
∴ The angle between the planes ABC and BFC is
109. 20
AC  cm
tan 60
15. (a) Consider △VBC. Consider △ABC.
8 By the cosine formula, we have
VB  cm
sin 40 AC 2  BC 2  AB 2
∵ △ABC is an equilateral triangle.
cos ACB 
2( AC )( BC )
8
∴ AB  BC  cm  20 
2
tan 40    30  (40 cos 30)
2 2

△ ACV  △ BCV (AAS)  tan 60 


∵ 
VA  VB (corr. sides,  △ s)  20 
∴ 2 (30)
∵ AM  MB and VA  VB  tan 60 
∴ VM  AB (prop. of isos. △) ACB  103.9197
AB  104 (cor. to 3 sig. fig.)
MB 
2
(b) The angle between the line HM and the plane ABC is
8 HMA.
 tan 40 cm Consider △AMC.
2
4
 cm
tan 40
224
6 Applications of Trigonometry in 3-dimensional Problems

By the cosine formula, we have

AM 2
 AC 2
 MC 2 18. (a) The angle between the line DC and the plane ABC is
 2( AC )( MC ) cos ACM

 20 
2 DCB.
   15 2
AM  tan 60   Consider △DCB. cm
 20 
 2  (15) cos 103.9197 DB
 21.0159 cm
 tan 60 
sin DCB 
DC
Consider △HMA.
HA 8 cm
tan HMA  
AM 10 cm
20 cm DCB  53.1 (cor. to 3 sig. fig.)
 ∴ The angle between the line DC and the plane ABC
21.0159 cm is 53.1°.
HMA  43.6 (cor. to 3 sig. fig.)
∴ The angle between the line HM and the plane ABC (b) ∵ △ ADB  △ CDB (RHS)
is 43.6°. ∴ BA  BC (corr. sides, △ s)
17. (a) The angle between the planes ABFE and ACDE is Let M be the mid-point of AC.
∠BAC (or∠FED) and the angle between the planes
ACDE and BCDF is∠ACB (or∠EDF).
∴ ∠BAC =∠FED = 100°
∠ACB =∠EDF = 30°
In △ABC,

∠ ABC ∠ ACB ∠ BAC  180 (∠ sum of △)


∠ ABC  30  100  180 ∵ △ADC and △ABC are isosceles triangles.
∠ ABC  50 ∴ DM  AC and BM  AC
Consider △ABC. ∴ The angle between the planes ADC and ABC is
By the sine formula, we have BMD.
Consider △DCB.
AB AC
 BC  10 2  8 2 cm (Pyth. theorem)
sin∠ ACB sin∠ ABC
AB 8 cm  6 cm
 Consider △BCM.
sin 30 sin 50
BM  BC sin 45
AB  5.2216 cm
 6 sin 45 cm
 5.22 cm (cor. to 3 sig. fig.) Consider △DBM.
DB
(b) Let N be the projection of M on the plane DEF. tan BMD 
The angle between the line MD and the plane DEF is BM
∠MDN. 8 cm
∵ M is the mid-point of AB. 
∴ 6sin45 cm
1 5.2216 cm BMD  62.1 (cor. to 3 sig. fig.)
AM  AB   2.6108 cm ∴ The angle between the planes ADC and ABC is
2 2
62.1°.
∴ EN  2.6108 cm
Consider △DEN. 19. (a) In △BCD,
By the cosine formula, we have BD 2  BC 2  CD 2 (Pyth. theorem)
BD  a  a cm
2 2
DN 2  ED 2  EN 2  2( ED)( EN ) cos∠ NED
 2 a cm
DN  8 2  2.61082  2(8)(2.6108) cos100 cm Let M be the mid-point of AF.
 8.8357 cm
Consider △DMN.
MN
tan∠ MDN 
DN
6 cm

8.8357 cm
∠ MDN  34.2 (cor. to 3 sig. fig.)
∴ The angle between the line MD and the plane
DEF is 34.2°.

225
NSS Mathematics in Action (2nd Edition) 5A Full Solutions

AN 2  AB 2  BN 2 (Pyth. theorem)
2
a
AN  a 2    cm
2
3
 a cm
2
3
FN  AN  a cm
2

1 2a
BM  BD  cm (property of square)
2 2
In △ABM,
AM 2  AB 2  BM 2 (Pyth. theorem)
2
 2 a 
AM  a 
2
cm
 2 

2
 a cm
2
AF  2 AM
2
 2 a cm
2
 2 a cm

(b) (i) The angle between the lines AB and BF is ABF.


Consider △ABF.
By the cosine formula, we have
BF 2  AB 2  AF 2
cos ABF 
2( BF )( AB )
a 2  a 2  ( 2 a) 2

2(a )(a )
0
∵ cos ABF is independent of a.
∴ The angle between the lines AB and BF, i.e.
ABF, will not change if the value of a
changes.
(ii) Let N be the mid-point of BC.

∵ BC is the line of intersection of the planes


ABC and FBC with AN  BC and FN  BC.
∴ The angle between the planes ABC and FBC
is ANF.

226
6 Applications of Trigonometry in 3-dimensional Problems

Consider △ANF. (ii) Let h cm be the shortest distance between the point
By the cosine formula, we have D and the plane ABC.
AN 2  FN 2  AF 2 Area of △ABC
cos ANF  1
2( AN )( FN )   AB  AC  sin∠ BAC
2 2 2
 3   3  1 
 a   a   ( 2a) 2    12  10  sin 60  cm 2
 2   2  2
     
 3  3   30 3 cm 2
2 a  a
 2  2  ∵ Volume of the tetrahedron
  
1  80.1866 cm 3
 ∴
3
∵ cos ANF is independent of a. 1
 30 3  h  80.1866
∴ The angle between the planes ABC and FBC, 3
i.e. ANF, will not change if the value of a
changes.
h  4.63 (cor. to 3 sig. fig.)
∴ The shortest distance between the point D and
20. (a) Consider △ABC. the plane ABC is 4.63 cm.
By the cosine formula, we have
21. (a) Let M be the mid-point of AB.
∵ VA = VB and AM = BM
BC 2
 AB  AC  2( AB )( AC ) cos∠ BAC
2 2
∴ VM⊥AB (prop. of isos. △)
BC  12 2  10 2  2(12)(10) cos 60 cm ∵ CA = CB and AM = BM
∴ CM⊥AB (prop. of isos. △)
 124 cm ∵ AB is the line of intersection of the planes ABV and
 11.1 cm (cor. to 3 sig. fig.) ABC with VM⊥AB and CM⊥AB.
∴ ∠VMC is the angle between the planes ABV and
Consider △ABD. ABC.
AD 2  DB 2  AB 2 (Pyth. theorem) In △VAM,
VM 2  AM 2  VA 2 (Pyth. theorem)
DB  12  7 2 cm
2

2
 95 cm 6
VM  62    cm
 9.75 cm (cor. to 3 sig. fig.) 2
Consider △ACD.  27 cm
AD 2  DC 2  AC 2 (Pyth. theorem) Similarly, CM  27 cm
DC  102  7 2 cm Consider △VMC.
By the cosine formula, we have
 51 cm
VM 2  CM 2  VC 2
 7.14 cm (cor. to 3 sig. fig.) cos∠ VMC 
2(VM )(CM )
BC  DB  DC ( 27 ) 2  ( 27 ) 2  6 2
(b) (i) Let s  . 
2 2( 27 )( 27 )
124  95  51 ∠ VMC  70.5288
i.e. s  cm .
2  70.5 (cor. to 3 sig. fig.)
Area of △BCD ∴ The angle between the planes ABV and ABC is
70.5°.
 s ( s  BC )( s  DB )( s  DC )
(b) Let N be the projection of V on the plane ABC.
 s( s  124 cm)( s  95 cm)( s  51 cm) The height of the tetrahedron is VN.
Consider △VMN.
 34.3657 cm 2 VN
Volume of the tetrahedron sin∠ VMN 
VM
1
  area of △ BCD  AD VN  27 sin 70.5288 cm
3
 4.90 cm (cor. to 3 sig. fig.)
1 
   34.3657  7  cm 3 ∴ The height of the tetrahedron is 4.90 cm.
3 
 80.1866 cm 3
 80.2 cm 3 (cor. to 3 sig. fig.)

227
NSS Mathematics in Action (2nd Edition) 5A Full Solutions

3. (a) (i) TCA  40


Consider △TAC.
Exercise 6C (p. 6.41)
Level 1 TA
tan TCA 
1. (a) ∵ DF is perpendicular to the ground BCFE. AC
∴ F is the projection of D on the ground BCFE. 500 m
∴ The projection of BD on the ground BCFE is BF. tan 40 
AC
BF  352  162 m (Pyth. theorem)
500
 1481 m AC  m
tan 40
 38.5 m (cor. to 3 sig. fig.)
 595.8768 m
(b) Consider △CDF.  596 m (cor. to 3 sig. fig.)
DF (ii) TBA  60
tan DCF  Consider △TAB.
FC
DF TA
tan 40  tan TBA 
16 m AB
500 m
DF  16 tan 40 m tan 60 
Consider △BFD. AB
BD 2  BF 2  DF 2 (Pyth. theorem) 500
AB  m
tan 60
BD  ( 1481 )  (16 tan 40) 2 m
2
 288.6751 m
 40.8 m (cor. to 3 sig. fig.)
 289 m (cor. to 3 sig. fig.)
∴ The distance the car travelled is 40.8 m.
(iii) Consider △ACB.
2. (a) Consider △ACB.
AB BC 2  AB 2  AC 2 (Pyth. theorem)
cos ABC 
BC 2 2
 500   500 
AB BC      m
cos 30   tan 60   tan 40 
100 m
 662 m (cor. to 3 sig. fig.)
AB  100 cos 30 m
Consider △TBA.
TA
tan TBA 
AB
TA
tan 20 
100 cos 30 m
TA  100 cos 30 tan 20 m
 31.5 m (cor. to 3 sig. fig.)
∴ The height of the tower TA is 31.5 m.

(b) Consider △ACB. (b)


AC
sin ABC  With the notation in the figure, consider △ACB.
BC
AC
AC tan ABC 
sin 30  AB
100 m
595.8768 m
AC  100 sin 30 m 
288.6751 m
 50 m
ABC  64.2 (cor. to 3 sig. fig.)
Consider △TCA.
TA BCP  ABC (alt. s, CP // AB )
tan TCA 
AC  64.2
100 cos 30 tan 20 m ∴ The compass bearing of B from C is N64.2W.

50 m 4. (a) Consider △ABC.
TCA  32.2 (cor. to 3 sig. fig.)
∴ The angle of elevation of T from C is 32.2. ACB  36  70  180 ( sum of △)
ACB  74
By the sine formula, we have

228
6 Applications of Trigonometry in 3-dimensional Problems

CA AB Consider △XAC.
 XC
sin CBA sin ACB tan XAC 
CA 200 m CA
 50 m
sin 70 sin 74 
200 sin 70 200 sin 70
CA  m m
sin 74 sin 74
XAC  14.3 (cor. to 3 sig. fig.)
∴ The angle of elevation of X from A is 14.3.

(b) Consider △ABC.


By the sine formula, we have
CB AB

sin CAB sin ACB
CB 200 m

sin 36 sin 74
200 sin 36
CB  m
sin 74
Consider △XBC.
XC
tan XBC 
CB
50 m

200 sin 36
m
sin 74
XBC  22.2 (cor. to 3 sig. fig.)
∵ The angle of depression of B from X
 the angle of elevation of X from B
∴ The angle of depression of B from X is 22.2.

5. Consider △PQR.
By the cosine formula, we have
PQ 2  QR 2  PR 2  2(QR )( PR ) cos∠ PRQ
PQ  300 2  100 2  2(300)(100) cos 55 m
 256.0965 m
Consider △TPQ.
TP
tan∠ TQP 
PQ
TP  256.0965 tan 20 m
 93.2115 m
Consider △TPR.
TP
tan∠ TRP 
PR
93.2115 m

100 m
∠ TRP  43.0 (cor. to 3 sig. fig.)
∴ The angle of elevation of T from R is 43.0°.

6. (a) Consider △ABP.


AB
tan 25 
150 m
AB  150 tan 25 m
 69.9 m (cor. to 3 sig. fig.)
∴ The height of the tower AB is 69.9 m.

(b) PBQ  150  90


 60

229
NSS Mathematics in Action (2nd Edition) 5A Full Solutions

By the sine formula, we have


BP PQ 8. (a) Consider △OAB.

sin BQP sin PBQ BA
tan AOB 
150 m 200 m OA

sin BQP sin 60 1000 m
tan 30 
BQP  40.5054 OA
1000
OA  m
BPQ  180  BQP  PBQ ( sum of △ ) tan 30
Consider △ODC.
 180  40.5054  60
CD
 79.4946 tan DOC 
Consider △BQP. OD
By the cosine formula, we have 1000 m
tan 20 
OD
BQ 2  BP 2  PQ 2  2( BP )( PQ) cos BPQ 1000
OD  m
BQ  150 2  200 2  2(150)( 200) cos 79.4946 m tan 20
 227.0690 m Consider △AOD.
Consider △ABQ. OA
cos AOD 
AB OD
tan AQB 
BQ 1000
m

69.9461 m
 tan 30 
227.0690 m 1000
m
AQB  17.1 (cor. to 3 sig. fig.) tan 20
∴ The angle of elevation of A from Q is 17.1. AOD  50.9 (cor. to 3 sig. fig.)
∴ The compass bearing of C from O is N50.9E.
7. Let h m be the height of the tower TO.
Consider △TAO.
(b) Distance travelled by the helicopter
hm  BC
tan 50 
OA  20v m
h Consider △AOD.
OA  m
tan 50 OD 2  OA 2  AD 2 (Pyth. theorem)
Consider △TBO.
 OA 2  BC 2
hm
tan 30  2
 1000   1000 
2
OB      (20v)
2

h  tan 20   tan 30  


OB  m 2
tan 30 1000 1000 2
400v 2  
AOB  90  40 tan 2 20 tan 2 30
 50  1 1 
Consider △OAB. v  2500   
 tan 20 tan 30 
2 2
By the cosine formula, we have
 107 (cor. to 3 sig. fig.)
AB 2  OA2  OB 2  2(OA)(OB ) cos AOB
2 2 9. (a) Consider △TAC.
 h   h  TC
350  
2
   tan TAC 
 tan 50    tan 30   AC
 h  h  50 m
 2   cos 50 tan 30 
 tan 50  tan 30  AC
 1 1 2 cos 50  2 50
122 500      h AC  m
tan 30
 tan 50 tan 30 tan 30 tan 50 
2 2

Consider △TBC.
122 500
h
1 1 2 cos 50
 
tan 2 50 tan 2 30 tan 30tan 50
 258 (cor. to 3 sig. fig.)
∴ The height of the tower TO is 258 m.

230
6 Applications of Trigonometry in 3-dimensional Problems

TC OA
tan TBC  cos OAC 
BC AC
50 m 20
tan 20  m
BC  tan 30
50 (30  15) m
BC  m
tan 20 OAC  39.6640
ACB  120  90  39.7 (cor. to 3 sig. fig.)
 30 ∴ The true bearing of C from A is 039.7.
Consider △ACB.
By the cosine formula, we have (b) Consider △OAC.
OC 2  AC 2  OA 2 (Pyth. theorem)
AB 2
 AC 2
 BC 2
 2( AC )( BC ) cos AC
2
 20 
2 2
  50  50 
    
AB   tan
 30

2
tan 20   OC  45    m
 2


50  
 
tan 30   tan
50
20 

 cos  tan 30  30

 75.9309 m
 75.9 m (cor. to 3 sig. fig.)  825 m
∴ The distance between A and B is 75.9 m. Consider △TCO.
TO
tan TCO 
OC
20 m

825 m
TCO  34.8 (cor. to 3 sig. fig.)
∴ The angle of elevation of T from C is 34.8.
(c) Consider △OAB.
By the cosine formula, we have
OB 2  OA 2  AB 2  2(OA)( AB ) cos OAB
(b)
2
 20   20 
With the notations in the figure, OB     30 2  2 (30) cos 39.6640
 tan 30    tan 30 
CBD  ACB (alt. s, DB // CA)
 22.3607 m
 30 Consider △TBO.
Consider △ACB.
By the cosine formula, we have TO
tan TBO 
BC  AB  AC
2 2 2 OB
cos CBA  20 m
2( BC )( AB) 
2 2 22.3607 m
 50  2  50  TBO  41.8 (cor. to 3 sig. fig.)
   75.9309   
tan 20   tan 30 
 ∴ The angle of elevation of T from B is 41.8.
 50 
2 (75.9309) 11. (a) (i) Consider △EPQ.
 tan 20  PQ
CBA  34.7692 tan∠ PEQ 
EQ
ABF  270  CBD  CBA
Reflex 3m
 270  30  34.7692 tan  
EQ
 335 (cor. to 3 sig. fig.)
3
∴ The true bearing of A from B is 335. EQ  m
tan 
10. (a) Consider △TAO. (ii) Consider △DSR.
TO SR
tan TAO  tan∠ SDR 
OA DR
20 m 2m
tan 30  tan  
OA DR
20 2
OA  m DR  m
tan 30 tan 
Consider △OAC.
(b) ∵ The area of the shadow = 10 3 m 2

231
NSS Mathematics in Action (2nd Edition) 5A Full Solutions

Consider △ABM.
 3 2 
4   AM 2  BM 2  AB 2 (Pyth. theorem)
 tan  tan    10 3
2 AM  13  5 2 cm
2

10  12 cm
∴  10 3 Consider △ABD.
tan 
∠ADB = 90° (prop. of isos. △)
1
tan   BD
3 cos∠ ABD 
AB
  30 BD
cos 50 
13 cm
(c) ED is the line of intersection of the plane PSDE and the
horizontal ground.
BD  13 cos 50 cm
∵ ∠ EDR  90 , i.e. ED is not perpendicular to  8.3562 cm
DR. AD
∴ θ is not the angle between the plane PSDE and sin∠ ABD 
the horizontal ground. AB
AD
12. (a) Let M be the mid-point of BC. sin 50 
13 cm
∵ DB = DC and BM = CM
∴ DM⊥BC (prop. of isos. △) AD  13 sin 50 cm
∵ AB = AC and BM = CM  9.9586 cm
∴ AM⊥BC (prop. of isos. △) Consider △BDM.
∵ BC is the line of intersection of the plane DBC and
the horizontal table with DM⊥BC and AM⊥BC. BM 2  DM 2  BD 2 (Pyth. theorem)
∴ The angle between the plane DBC and the
horizontal table is∠AMD. DM  8.3562 2  5 2 cm
10  6.6952 cm
BM  cm  5 cm Consider △ADM.
2
AM 2  DM 2  AD 2
cos∠ AMD 
2( AM )( DM )
12 2  6.6952 2  9.9586 2

2(12)(6.6952)
∠ AMD  56.1 (cor. to 3 sig. fig.)
∴ The angle between the plane DBC and the
horizontal table is 56.1°.

(b) ∵ AD is the line of intersection of the planes ADB


and ADC with AD⊥BD and AD⊥CD.
∴ ∠BDC is the angle between the planes ADB and
ADC.
∴ The student is correct.

Level 2
13. (a) (i) Consider △EBC.
BE  8 2  6 2 m (Pyth. theorem)
 10 m
(ii) Consider △BCD.
DC
tan 35 
6m
DC  4.2012 m
EF  DC
 4.20 m (cor. to 3 sig. fig.)
(iii) Consider △EBF.

BF  BE 2  EF 2 (Pyth. theorem)
 10 2  4.2012 2 m
 10.8467 m
 10.8 m (cor. to 3 sig. fig.)

232
6 Applications of Trigonometry in 3-dimensional Problems

AG 2  BG 2  AB 2 (Pyth. theorem)
2 2
 h   h 
(b) Consider △FDB.      4502
FD  tan 25   tan 15 
sin FBD 
BF 4502
h
8m 1 1
 2
 2
10.8467 m tan 25 tan 15
FBD  47.5 (cor. to 3 sig. fig.)  105 (cor. to 3 sig. fig.)
∴ The angle of inclination of FB is 47.5.

(c) ∵ G is the mid-point of EF.


1
GE  EF
2

4.2012
 m
2
 2.1006 m
Consider △GBE.
GB 2  GE 2  BE 2 (Pyth. theorem) (b)
GB  2.1006  10 2 2
m
With the notation in the figure,
 10.2182 m
AG
Let H be the projection of G on the plane ABCD. tan ABG 
BG
h
m
 tan 25
h
m
tan 15
tan 15

Consider △BGH.
tan 25
GH ABG  29.8825
sin GBH 
GB
HAB  ABG (alt. s, AH // GB )
8m
  29.9 (cor. to 3 sig. fig.)
10.2182 m
∴ The compass bearing of B from A is S29.9E.
GBH  51.5 (cor. to 3 sig. fig.)
∴ The angle of inclination of GB is 51.5. 15. (a) (i) ∠BAC = 85° – 38° = 47°
Consider △ABC.
14. (a) (i) Consider △TAG. By the cosine formula, we have
TG
tan TAG 
AG BC 2  AB 2  AC 2  2( AB)( AC ) cos∠ BAC
hm BC  70 2  712  2(70)(71) cos 47 m
tan 25 
AG  56.2311 m
h
AG  m  56.2 m (cor. to 3 sig. fig.)
tan 25
Consider △TBG.
TG
tan TBG 
BG
hm
tan 15 
BG
(ii)
h
BG  m
tan 15 With the notations in the figure,
By the cosine formula, we have
(ii) Consider △ABG.

233
NSS Mathematics in Action (2nd Edition) 5A Full Solutions

AC 2  BC 2  AB 2 Consider △BCX.
cos∠ ACB  XC
2( AC )( BC ) tan∠ XBC 
BC
712  56.23112  70 2 71 tan 35 m
 
2(71)(56.2311) 56.2311 m
∠ ACB  65.5655 ∠ XBC  41.5 (cor. to 3 sig. fig.)
∠ACP =∠CAQ = 38° (alt.∠s, CP // QA)
∴ The angle of elevation of X from B is 41.5°.
∠ BCR  180  (65.5655  38)
 152 (cor. to 3 sig. fig.)
∴ The true bearing of B from C is 152°.

(b) Consider △ACX. 16. (a)


XC
tan 35  With the notation in the figure,
71 m Let AR be the length of Tom’s shadow.
XC  71tan 35 m 40 m  AR 14 m

AR 1.5 m (corr. sides, ~△s)
40 m 14
 1
AR 1.5
AR  4.8 m
∴ The length of Tom’s shadow is 4.8 m.

(b) (i)

With the notations in the figure,

FDO  TFG (corr. s, OD // GF )


 28
Consider △CDF.
1.5 m
tan 28 
CD
CD  2.82 m (cor. to 3 sig. fig.)
∴ The length of his shadow CD is 2.82 m.
(ii) The shortest length of Tom’s shadow is attained
when the distance between the lamppost and Tom
is the shortest.
Suppose Tom stands at a point G on AB with
GO  AB .
Consider △OAB.
40 m
tan ABO 
25 m
ABO  57.9946
Consider △OGB.
OG
sin OBG 
OB
OG
sin 57.9946 
25 m
OG  21.2000 m
Let GH be the length of Tom’s shadow when he is
at G.

234
6 Applications of Trigonometry in 3-dimensional Problems

21.2 m  GH 14 m (b) Let C be a point on QR such that PC⊥QR.


 Then, PC is the shortest distance between P and QR.
GH 1.5 m ∴ The greatest angle of elevation of T along QR is
21.2 m 14 (corr. sides, ∠TCP.
 1 Consider △CPQ.
GH 1.5 PC
GH 1.5 sin∠ PQC 
 PQ
21.2 m 12.5
~△s) PC  100 3 sin 19.2103 m
 56.9908 m
GH  2.54 m (cor. to 3 sig. fig.) Consider △TPC.
TP
∴ The shortest length of Tom’s shadow is tan∠ TCP 
2.54 m. PC
∴ I agree with Tom’s claim. 100 m

56.9908 m
∠ TCP  60.3 (cor. to 3 sig. fig.)
∴ It is impossible for Vicky to find a point X on QR
such that the angle of elevation of T from X is
greater than 65°.

18. (a) (i) Consider △PAO.


PO
tan PAO 
17. (a) (i) OA
hm
With the notations in the figure, tan 30 
OA
∠ APR  ∠ QPR  180 (adj.∠ s on st. line) h
OA  m
100  ∠ QPR  180 tan 30
∠ QPR  80  3h m
Consider △TPQ. ∴ The distance between O and A is 3h m .
100 m (ii) Consider △PBO.
tan 30 
PQ PO
tan PBO 
PQ  100 3 m OB
Consider △TPR. hm
tan 60 
100 m OB
tan 60 
PR h
100 OB  m
PR  m tan 60
3 h
Consider △PQR.
 m
3
By the cosine formula, we have
h
∴ The distance between O and B is m.
  
3
2 2 2
QR PQ PR 2( PQ )( PR ) cos∠ QPR
2
 100 
  
(iii) Consider △PCO.
2
(100   3 )
 3 
QR  m
 100 
 2(100 3 )

 cos 80
 PO
tan PCO 
 3 
 172.8015 m
 173 m (c or. to 3 sig. fig.) OC
(ii) By the cosine formula, we have hm
PQ 2  QR 2  PR 2 tan 45 
cos∠ PQR  OC
2( PQ)(QR) h
2 OC  m
 100  tan 45
(100 3 )  172.8015  
2 2

 3  hm
 ∴ The distance between O and C is h m.
2(100 3 )(172.8015)
∠ PQR  19.2103
 19.2 (cor. to 3 sig. fig.)
∴ The compass bearing of R from Q is
N19.2°E.

235
NSS Mathematics in Action (2nd Edition) 5A Full Solutions

(b) (i) Consider △OAB.


By the cosine formula, we have
OA 2  AB 2  OB 2
cos OAB  QC  (5  2) cm
2(OA)( AB ) 19. (a)
2  3 cm
 h  Consider △QBC in Figure (b).
( 3h)  60  2
 2

3 
By the cosine formula, we have
 
2( 3h)(60)
BC 2  BQ 2  QC 2  2( BQ)(QC ) cos BQC
2
h
3h 2  3600  BC  2 2  32  2( 2)(3) cos 70 cm
 3
 2.9826 cm
120 3h
 2.98 cm (cor. to 3 sig. fig.)
h 2  1350

45 3h (b) Consider △QBC in Figure (b).
By the cosine formula, we have
(ii) Consider △OAC.
By the cosine formula, we have BQ 2  BC 2  QC 2
cos QBC 
OA2  AC 2  OC 2 2( BQ)( BC )
cos OAC 
2(OA)( AC ) 2 2  2.98262  32

( 3h) 2  100 2  h 2 2( 2)(2.9826)
 QBC  70.9403
2( 3h)(100)
 70.9 (cor. to 3 sig. fig.)
3h  10 000  h 2
2

200 3h (c) Let D be a point on BC such that PD  BC and
h  5000
2
QD  BC.

100 3h

(c) ∵ OAB = OAC


∴ cos OAB  cos OAC
h 2  1350 h 2  5000

45 3h 100 3h
20h  27 000  9h 2  45 000
2
The angle between the planes BCP and BCQ is PDQ.
11h 2  18 000 Consider △PQB.
h  40.4520 PQ
tan PBQ 
 40.5 (cor. to 3 sig. fig.) BQ
PQ
OA  3h m tan 60 
2 cm
 3 ( 40.4520) m PQ  2 tan 60 cm
 70.0649 m Consider △QBD.
OC  h m QD
sin QBD 
 40.4520 m QB
Consider △OAC.
By the cosine formula, we have QD
sin 70.9403 
OA 2  OC 2  AC 2 2 cm
cos AOC  QD  2 sin 70.9403 cm
2(OA)(OC )
Consider △PQD.
70.0649 2  40.4520 2  100 2 PQ
 tan PDQ 
2(70.0649)(40.4520) QD
AOC  127.5480 2 tan 60 cm
180  AOC  180  127.5480 
2 sin 70.9403 cm
 52.5 (cor. to 3 sig. fig.) PDQ  61.4 (cor. to 3 sig. fig.)
∴ The compass bearing of C from O is N52.5E. ∴ The angle between the planes BCP and BCQ is
61.4.

236
6 Applications of Trigonometry in 3-dimensional Problems

k
2 4 (8  r )( r  2)  14 cm 3
AP  k    cm (Pyth. theorem)
2

20. (a) 2  56 (8  r )(r  2) cm 3

3
 k cm
2

k
(b) (i) BP  PC  cm
2
 △PBC is an equilateral triangle.
k
∴ BC  2 cm

(ii) Let D be a point on BC such that AD  BC and


PD  BC .
The angle between the plane ABC and the
horizontal table is ADP.

Consider △BPD.
PD  PB cos 30
k 3
  cm
2 2
3k
 cm
4
Consider △ADP.

AP
tan ADP 
PD
3k
cm
 2
3k
cm
4
2
ADP  63.4 (cor. to 3 sig. fig.)
∴ The angle between the plane ABC and the
horizontal table is 63.4, which is a constant.
∴ Ken’s claim is correct.

PM  QM  PQ
21. (a) Let s  .
2
r  (10  r )  6
i.e. s cm  8 cm
2
Area of △MPQ
 s ( s  PM )( s  QM )( s  PQ )
 8(8  r )[8  (10  r )](8  6) cm 2
 4 (8  r )( r  2) cm 2
Volume of the prism

237
NSS Mathematics in Action (2nd Edition) 5A Full Solutions

(b) (i) Consider △MPQ.


By the cosine formula, we have

PQ 2  PM 2  QM 2  2( PM )(QM ) cos∠ PMQ


6 2  r 2  (10  r ) 2  2r (10  r ) cos 60
36  r 2  100  20r  r 2  10r  r 2
0  3r 2  30r  64
r  3.0851 or 6.9149
 3.09 (cor. to 3 sig. fig.) or
(ii)
6.91 (cor. to 3 sig. fig.)
(ii) Volume of the prism With the notations in the figure,
 56 (8  3.0851)(3.0851  2) cm 3

∠ XAO  ∠ AOY  20 (alt.∠ s, AX // PO)


 129 cm3 (cor. to 3 sig. fig.)
Consider △OAB.
By the cosine formula, we have
22. (a) (i) Consider △TAO.
OA 2  AB 2  OB 2
TO cos∠ OAB 
tan∠ TAO  2(OA)( AB )
OA
2
hm  122.8754 
tan 30  (122.8754 3 ) 2  200 2   
OA  3 

h 2(122.8754 3 )(200)
OA  m
tan 30 ∠ OAB  19.4704
 3h m ∠ XAB  20  19.4704
∴ The distance between O and A is 3h m .  39.5 (cor. to 3 sig. fig.)
(ii) Consider △TBO. ∴ The compass bearing of B from A is N39.5°E.
TO
tan∠ TBO  (c) (i) Consider △AOP.
OB
hm ∠ AOP ∠ APO ∠ OAP  180 (∠ sum of △)
tan 60 
OB ∠ APO  180  20  19.4704
h
OB  m  140.5296
tan 60 By the sine formula, we have
h OP OA
 m 
3 sin∠ OAP sin∠ APO
h 122.8754 3 sin 19.4704
∴ The distance between O and B is m. OP  m
3 sin 140.5296
 111 .5960 m
(b) (i) Consider △OAB. Consider △TOP.
∠ AOB  20  50  70 TO
By the cosine formula, we have tan∠ TPO 
OP
122.8754 m
AB 2  OA2  OB 2  2(OA)(OB) cos∠ AOB tan  
2
111 .5960 m
 h   h    47.8 (cor. to 3 sig. fig.)
200 2  ( 3h) 2     2( 3h)


 3  cos 70
 3  
 10 
40 000    2 cos 70  h 2
 3 
40 000
h 
2
10
 2 cos 70
3
h  122.8754 or  122.8754 (rejected)
 123 (cor. to 3 sig. fig.)
(ii)

With the notations in the figure,

238
6 Applications of Trigonometry in 3-dimensional Problems

∵ The angle of elevation of T from Q is . AG 2  AH 2  HG 2 (Pyth. theorem)


∴ OP = OQ
∴ ∠OPQ =∠OQP (base∠s, isos. △) AG  ( 32 ) 2  4 2 cm
 48 cm
OPA  OPQ  180 (adj. s on st. line)  6.93 cm (cor. to 3 sig. fig.)
OPQ  180  140.5296
 39.4704 (b) ∵ AH is the projection of AG on the plane ADHE.
∠ ZOQ  ∠ OPQ  ∠ OQP ∴ The angle between the line AG and the plane
ADHE is∠GAH.
 39.4704  39.4704
 78.9 (cor. to 3 sig. fig.)
∴ The compass bearing of Q from O is
N78.9°E.

Check Yourself (p. 6.50)


1. (a)  (b) 
(c)  (d) 
(e) 

2. ∠VAO

3. ∠VMO

4. VO

5. (a) (i)

AC  4 2  3 2 cm (Pyth. theorem)
 5 cm
(ii) The angle between the lines AG and AC is∠CAG.
Consider △ACG.
CG
tan∠ CAG 
AC
3 cm

5 cm
∠ CAG  31.0 (cor. to 3 sig. fig.)
∴ The angle between the lines AG and AC is
31.0°.

(b) (i)

AF  4 2  3 2 cm (Pyth. theorem)
 5 cm
(ii) The angle between the line AG and the plane
ABFE is∠FAG.
Consider △AFG.
FG
tan∠ FAG 
AF
3 cm

5 cm
∠ FAG  31.0 (cor. to 3 sig. fig.)
∴ The angle between the line AG and the plane
ABFE is 31.0°.

AH  4 2  4 2 cm (Pyth. theorem)
6. (a)
 32 cm
 5.66 cm (cor. to 3 sig. fig.)

239
NSS Mathematics in Action (2nd Edition) 5A Full Solutions

Consider △AGH. Consider △TOB.


AH TO
cos∠ GAH  tan∠ TBO 
AG OB
32 cm TO
 tan 55 
48 cm 120 sin 40 m
∠ GAH  35.3 (cor. to 3 sig. fig.) TO  120 sin 40 tan 55 m
∴ The angle between the line AG and the plane  110 m (cor. to 3 sig. fig.)
ADHE is 35.3°. ∴ The height of the tower TO is 110 m.

7. (a) ∵ AB is a line of greatest slope of the hillside. (b) Consider △OAB.


∴ The angle between a line of greatest slope of the
OA
hillside and the horizontal plane BCFE is∠ABE. cos∠ OAB 
Consider △ABC. AB
AB OA
tan∠ ACB  cos 40 
BC 120 m
AB OA  120 cos 40 m
tan 58  Consider △TOA.
32 m
AB  32 tan 58 m TO
tan∠ TAO 
Consider △ABE. OA
AE 120 sin 40 tan 55 m
sin∠ ABE  
AB 120 cos 40 m
12 m ∠ TAO  50.2 (cor. to 3 sig. fig.)

32 tan 58 m ∴ The angle of elevation of T from A is 50.2°.
∠ ABE  13.6 (cor. to 3 sig. fig.) Revision Exercise 6 (p. 6.52)
∴ The angle between a line of greatest slope of the Level 1
hillside and the horizontal plane BCFE is 13.6°. 1. (a) The angle between the lines AF and FG is∠AFG.
∵ ABFE and EFGH are perpendicular to each other.
(b) ∵ EC is the projection of AC on the horizontal plane ∴ ∠AFG = 90°
BCFE. ∴ The angle between the lines AF and FG is 90.
∴ The angle between the line AC and the plane
BCFE is∠ACE. (b) The angle between the lines AH and HC is∠AHC.
Consider △ABC.
AH  12  12 cm (Pyth. theorem)
BC
cos∠ ACB   2 cm
AC
32 m Similarly, AC  2 cm and CH  2 cm
cos 58  ∴ △ACH is an equilaterial triangle.
AC ∴ ∠AHC = 60°
32 ∴ The angle between the lines AH and HC is 60°.
AC  m
cos 58 (c) ∵ AB is the projection of AF on the plane ABCD.
Consider △ACE. ∴ The angle between the line AF and the plane
AE ABCD is∠BAF.
sin∠ ACE  BF
AC tan∠ BAF 
12 m AB
 1 cm
32 
m
cos 58 1 cm
∠ ACE  11 .5 (cor. to 3 sig. fig.) ∠ BAF  45
∴ The angle between the line AC and the horizontal ∴ The angle between the line AF and the plane
plane BCFE is 11.5°. ABCD is 45°.

8. (a) Consider △OAB. (d) ∵ AD is the line of intersection of the planes AHD
and ABCD with HD⊥AD and CD⊥AD.
OB ∴ The angle between the planes AHD and ABCD is
sin∠ OAB 
AB ∠CDH, i.e. 90.
OB
sin 40  2. (a) Consider △AGF.
120 m AG 2  AF 2  FG 2 ( Pyth. theorem)
OB  120 sin 40 m
AG  4 2  6 2 cm
 7.21 cm (cor. to 3 sig. fig.)

240
6 Applications of Trigonometry in 3-dimensional Problems

Consider △GHE. Consider △NPX.


EG 2  EH 2  HG 2 ( Pyth. theorem) PN 2  PX 2  NX 2 (Pyth. theorem)
EG  6  3 cm
2 2
PN  5  8 cm
2 2

 45 cm  9.43 cm (cor. to 3 sig. fig.)


 6.71 cm (cor. to 3 sig. fig.) ∴ The shortest distance between the point P and the
line AB is 9.43 cm.
(b) ∵ AF is perpendicular to the plane EFGH.
∴ The shortest distance between the point A and the
plane EFGH is AF, i.e. 4 cm.

(c) Let M be a point on EG such that FM⊥EG.


The shortest distance between the point F and the line
EG is FM.
1  (b)
Area of △EFG
   3  6  cm 2
 2  Let Q be the projection of P on the plane ABCD.
 9 cm 2 The angle between the line PA and the base ABCD is
PAQ.
1 Consider △ABC.
Also, area of △ EFG   GE  FM
2 AC 2  AB 2  BC 2 (Pyth. theorem)
1
9 cm   ( 45 cm)  FM
2
AC  12 2  16 2 cm
2
FM  2.68 cm (cor. to 3 sig. fig.)  20 cm
∴ The shortest distance between the point F and the 1
AQ  AC (property of rectangle)
line EG is 2.68 cm. 2
1
3. (a) The angle between the lines BD and BA is ABD.   20 cm
Consider △ABD. 2
AD  10 cm
tan ABD  Consider △PQA.
AB
PQ
50 cm tan PAQ 
 AQ
40 cm
5 cm
ABD  51.3 (cor. to 3 sig. fig.) 
10 cm
∴ The angle between the lines BD and BA is 51.3.
PAQ  26.6 (cor. to 3 sig. fig.)
(b) The angle between the line BD and the horizontal plane ∴ The angle between the line PA and the base ABCD
BCEF is DBE. is 26.6.
Consider △CDE.
DE 5. ∵ △DEF is an equilateral triangle.
sin 10  ∴ DFE  60
40 cm
Let M be a point on EF such that XM  EF and DM  EF.
DE  40 sin 10 cm
Consider △ABD.
BD  40 2  50 2 cm (Pyth. theorem)
 4100 cm
Consider △BDE.
DE
sin DBE 
BD
40 sin 10 cm

4100 cm Consider △DFM.
DBE  6.23 (cor. to 3 sig. fig.) DM 2  DF 2  MF 2 ( Pyth. theorem)
∴ The angle between the line BD and the horizontal
DM  8  4 cm
2 2
plane BCEF is 6.23.
 48 cm
4. (a) Let N be a point on AB such that PN⊥AB. Consider △XDM.
The shortest distance between the point P and the line
AB is PN.
Let X be the projection of P on the plane ABCD.
NX  8 cm and PX  5 cm

241
NSS Mathematics in Action (2nd Edition) 5A Full Solutions

DX 30
tan XMD  ∠RPM =  15
DM 2
DX Consider △MPR.
tan 36 
48 cm RM
sin∠ RPM 
DX  48 tan 36 cm PR
AX  AD  DX
120
cm
 (8  48 tan 36) cm sin 15  2
PR
 2.97 cm (cor. to 3 sig. fig.)
60
PR  cm
6. (a) Consider △ABC. sin 15
ABC  ACB  BAC  180  231.8222 cm
(
60  72  BAC  180  232 cm (cor. to 3 sig. fig.)
BAC  48
sum of △) (b) Consider △DPR.
PD 2  DR 2  PR 2 (Pyth. theorem)
By the sine formula, we have
AB BC PD  231.8222 2  210 2 cm
  98.1913 cm
sin ACB sin BAC
AB 14 cm  98.2 cm (cor. to 3 sig. fig.)

sin 72 sin 48
(c) ∵ PD is perpendicular to the horizontal ground and
14 sin 72
AB  cm PQ = PR.
sin 48 ∴ The length of the projection of PQ on the ground
 17.9168 cm = the length of the projection of PR on the ground
i.e. DQ = DR
 17.9 cm (cor. to 3 sig. fig.) ∵ DQ = DR and QM = RM
Consider △PBA. ∴ DM⊥QR (prop. of isos. △)
∵ QR is the line of intersection of the plane PQR and
PA the horizontal ground with PM⊥QR and
tan PBA 
AB DM⊥QR.
PA ∴ The angle between the plane PQR and the
tan 30  horizontal ground is∠PMD.
17.9168 cm Consider △MPR.
PA  10.3443 cm MR
 10.3 cm (cor. to 3 sig. fig.) tan∠ RPM 
PM
60 cm
(b) Area of △ABC tan 15 
PM
1 60
  AB  BC  sin ABC PM  cm
2 tan 15
1  223.9230 cm
  17.9168  14  sin 60 cm 2 Consider △DMP.
2
 108.6148 cm 2 PD
sin∠ PMD 
PM
 109 cm 2 (cor. to 3 sig. fig.)
98.1913 cm

(c) Volume of the tetrahedron PABC 223.9230 cm
1 ∠ PMD  26.0 (cor. to 3 sig. fig.)
  area of △ ABC  PA ∴ The angle between the thin metallic sheet PQR and
3
the horizontal ground is 26.0°.
1
  108.6148  10.3443 cm 3
3 8. (a) (i) One of the longest line segments is AG.
In △ABC,
 375 cm 3 (cor. to 3 sig. fig.)
AC  2 2  32 m (Pyth. theorem)
7. (a) Let M be the mid-point of QR.  13 m
Join PM. In △AEG,
∵ PQ  PR and QM  RM
∴ ∠QPM =∠RPM and PM⊥QR
(prop. of isos. △) AG  ( 13 ) 2  5 2 m (Pyth. theorem)
 6.16 m (cor. to 3 sig. fig.)
(ii) The angle between the line AG and the plane
242
6 Applications of Trigonometry in 3-dimensional Problems

ABCD is GAC. Consider △BAP.


CG BPA = 200 150
tan GAC  = 50
AC By the cosine formula, we have
5m
   
13 m
2 2 2
AB BP AP 2
2
 45 
  
GAC  54.2 (cor. to 3 sig. fig.) AB 
 45
 tan

40 

2 
40 
∴ The required angle is 54.2. 
 tan
59.811 4 m
 59.8 m (cor. to

(b) (i)

One of the longest line segments is AV.


In △ABC,
AC  62  6 2 cm (Pyth. theorem)
 72 cm (b)
With the notation in the figure,
1 With the notations in the figure,
AO  AC (property of square) BPQ  200  180  20
2
RBP  BPQ (alt. s, RB // PQ )
72
 cm  20
2
Consider △BAP.
In △OAV,
By the cosine formula, we have
BP 2  AB 2  AP 2
 72 
2
cos PBA 
AV     82 cm (Pyth. theorem) 2( BP)( AB )
 2 
  2 2
 45  2  45 
 9.06 cm (cor. to 3 sig. fig.)    59.8114   
 tan 40   tan 30 
(ii) The angle between the line AV and the plane 
ABCD is VAO.  45 
2 (59.8114)
VO  tan 40 
tan VAO 
AO PBA  86.6179
8 cm RBA  RBP  PBA

72  20  86.6179
cm
2  107 (cor. to 3 sig. fig.)
VAO  62.1 (cor. to 3 sig. fig.) ∴ The true bearing of A from B is 107.
∴ The required angle is 62.1. 10. (a) Consider △ABP.
9. (a) Consider △TBP. PB
tan BAP 
TP AB
tan TBP  PB
BP tan 50 
45 m 2m
tan 40  PB  2 tan 50 m
BP
 2.38 m (cor. to 3 sig. fig.)
45
BP  m
tan 40
Consider △TAP.
TP
tan TAP 
AP
45 m
tan 30  (b)
AP
45 Consider △ABD.
AP  m
tan 30

243
NSS Mathematics in Action (2nd Edition) 5A Full Solutions

3m (c) (i) Consider △BCP.


tan ABD 
2m
ABD  56.3099 PB
tan BCP 
With the notations in the figure, BC
2 tan 50 m

KBD  56.3099  15 3m
 41.3 (cor. to 3 sig. fig.) BCP  38.5 (cor. to 3 sig. fig.)
BDM  KBD (alt. s, DM // KB ) ∴ The angle of elevation of P from C is 38.5°.
(ii) In △BCD,
 41.3
∴ The compass bearing of P from D is S41.3°W. BD  32  2 2 m (Pyth. theorem)
 13 m
Consider △BDP.

PB
tan BDP 
BD
2 tan 50 m

13 m
BDP  33.5 (cor. to 3 sig. fig.)
∴ The angle of elevation of P from D is 33.5°.

11. (a)

With the notations in the figure,


AG  BH  800 m
Consider △ACG.
ΑG
tan ACG 
CG
800 m
tan 50 
CG
800
CG  m
tan 50
Consider △CHG.
GH
tan HCG 
CG
GH
tan 70 
800
m
tan 50
800 tan 70
GH  m
tan 50
∴ The speed of the aeroplane
800 tan 70
m
 tan 50
10 s
 184 m/s (cor. to 3 sig. fig.)

244
6 Applications of Trigonometry in 3-dimensional Problems

(b) Consider △CHG. ABCD is∠VAN, i.e. ∠VAN = .


CG
cos HCG 
CH
800
m
cos 70  tan 50
CH
800
CH  m
tan 50 cos 70
Consider △BCH.
BH
tan BCH 
CH
800 m

800
m
tan 50 cos 70
BCH  22.2 (cor. to 3 sig. fig.)
∴ The angle of elevation of B from C is 22.2°.

12. (a) Area of △PQR


1
  PQ  QR  sin PQR
2
1
  20  21  sin 36 cm 2
2
 123.4349 cm 2
 123 cm 2 (cor. to 3 sig. fig.)

(b) Let M be a point on QR such that PM⊥QR and


TM⊥QR.
The shortest distance between the point T and the line
QR is MT.

1
Area of △ PQR   QR  PM
2
1
123.4349 cm 2   21 cm  PM
2
PM  11 .7557 cm
Consider △PMT.
PM 2  PT 2  MT 2 (Pyth. theorem)
MT  11 .7557 2  102 cm
 6.18 cm (cor. to 3 sig. fig.)
∴ The shortest distance between the point T and the
line QR is 6.18 cm.

13. (a) (i) Consider △ABC.

AC 2  AB 2  BC 2 (Pyth. theorem)
AC  8 8
2 2
cm
 128 cm (or 8 2 cm)
(ii) Let N be the projection of V on the plane ABCD.
∵ VN is perpendicular to the plane ABCD.
∴ VN is the height of the pyramid.
Also, the angle between the line VA and the plane

245
NSS Mathematics in Action (2nd Edition) 5A Full Solutions

Consider △VAN.  5  area of △ AOB


128 1 2.5
AN  cm  5  5  sin 54 cm 2
2 2 cos 54
 43.0119 cm 2
VN
tan∠ VAN 
AN
VN
tan  
128
cm
2
128 tan 
VN  cm (or 4 2 tan cm)
2

(b) ∵ Volume of the pyramid = 144 cm3


1 128 tan 
 8 8  144
3 2
∴ 27
tan  
2 128
  50.0(cor. to 3 sig. fig.)

14. (a) With the notations in the figure,

 5  2)180 1
    54
5 2
Consider △AOX.
2.5 cm
cos  
OA
2.5
OA  cm
cos 54
 4.25 cm (cor. to 3 sig. fig.)

(b)

With the notations in the figure,


VA 2  VO 2  OA2 (Pyth. theorem)
2
 2.5 
VO  12 2    cm
 cos 54 
 11.2210 cm
Area of base ABCDE

246
6 Applications of Trigonometry in 3-dimensional Problems

Volume of the right pyramid formed ADC.


1
  area of base ABCDE  VO
3
1
  43.0119  11 .2210 cm3
3
 161 cm3 (cor. to 3 sig. fig.)

15. (a) Consider △DCE.


DE
sin 20 
CD
DE
CD 
sin 20

(b)

Let Q be the projection of P on the plane BCEF.


The angle between CP and the plane BCEF is PCQ.
Consider △PCD.
CD
cos PCD 
PC
DE
cos 50  sin 20
PC
DE
PC 
sin 20 cos 50
Consider △PCQ.
PQ
sin PCQ 
PC
DE

DE
sin 20 cos 50
PCQ  12.7 (cor. to 3 sig. fig.)
∴ The angle between CP and the plane BCEF is
12.7°.

16. (a) Consider △ABD.


AD 2  BD 2  (16 2  12 2 ) cm 2
 400 cm 2
AB 2  ( 20 cm) 2  400 cm 2
∵ AD 2  BD 2  AB 2
∴ ∠ADB = 90° (converse of Pyth. theorem)
Consider △ACD.
AD 2  CD 2  (16 2  632 ) cm 2
 4225 cm 2
AC  (65 cm) 2  4225 cm 2
2

∵ AD 2  CD 2  AC 2
∴ ∠ADC = 90° (converse of Pyth. theorem)
∵ AD is the line of intersection of the planes ADB
and ADC with BD⊥AD and CD⊥AD.
∴ ∠BDC is the angle between the planes ADB and

247
NSS Mathematics in Action (2nd Edition) 5A Full Solutions

(b) Consider △BCD. BD 2  AD 2  AB 2 (Pyth. theorem)


BD  CD  (12  63 ) cm
2 2 2 2 2
18. (a) (i)
BD  10  10 cm
2 2

 4113 cm 2
 200 cm
BC 2  (60 cm)2  3600 cm 2
1
∵ BD 2  CD 2  BC 2 OB  BD (property of square)
2
∴ ∠ BDC  90
200
∴ ∠ADB is not the angle between the planes CDB  cm
and CDA. 2
In △VOB,
17. (a) (i) Consider △ADF. VB 2  OB 2  VO 2 (Pyth. theorem)
DF
sin∠ DAF   200 
2

AD VB     82 cm
 2 
DF  
sin 30 
4m  114 cm
DF  2 m  10.7 cm (cor. to 3 sig. fig.)
HK  DF  2 m (ii) Consider △VAB.
Consider △AHK. VA  VB  114 cm
HK By the cosine formula, we have
sin∠ HAK 
AH VB 2  AB 2  VA 2
cos VBA 
2m 2(VB)( AB)
sin 26 
AH ( 114 ) 2  102  ( 114 ) 2
2 
AH  m 2( 114 )(10)
sin 26 VBA  62.0765
 4.5623 m
 62.1 (cor. to 3 sig. fig.)
 4.56 m (cor. to 3 sig. fig.)
Consider △BHK. AN
HK sin VBA 
sin∠ HBK  AB
BH (b) AN  10 sin 62.0765 cm
2m  8.8357 cm
sin 20 
BH  8.84 cm (cor. to 3 sig. fig.)
2
BH  m
sin 20 (c) ∵ △VAB  △VCB
 5.8476 m ∴ CN  VB and CN = AN
The angle between the planes VAB and VBC is
 5.85 m (cor. to 3 sig. fig.) ANC.
(ii) Consider △ABH. AC  BD
By the cosine formula, we have
 200 cm
AH 2  BH 2  AB 2
cos∠ AHB  Consider △NAC.
2( AH )( BH ) By the cosine formula, we have
4.5623 2  5.8476 2  5 2 AN 2  CN 2  AC 2
 cos ANC 
2(4.5623)(5.8476) 2( AN )(CN )
∠ AHB  55.7767 8.8357 2  8.8357 2  ( 200 ) 2

 55.8 (cor. to 3 sig. fig.) 2(8.8357)(8.8357)
ANC  106 (cor. to 3 sig. fig.)
(b) Time required for the blue toy car to reach H
∴ The angle between the planes VAB and VBC is
4.5623 m 106°.

0.3 m/s
 15.2 s (cor. to 3 sig. fig.) 19. (a) Let E be a point on CD such that AE  CD and
BE  CD.
Time required for the green toy car to reach H
5.8476 m

0.4 m/s
 14.6 s (cor. to 3 sig. fig.)
∴ The two toy cars will not reach H at the same time.

Level 2

248
6 Applications of Trigonometry in 3-dimensional Problems

BF 2
 FE 2
 BE 2
 2( FE )( BE
2
 155 
   ( 55 ) 2
 2 
BF   
 155 
 2 ( 55 ) cos
 2 
 
 6.2249 cm

The angle between the planes ACD and BCD is AEB.


1
CE  CD
2
6
 cm
2
 3 cm
In △BCE,
BE 2  BC 2  CE 2 (Pyth. theorem)
BE  8  3 cm
2 2

 55 cm
Consider △ABE.
AB
tan AEB 
BE
10 cm

55 cm
AEB  53.4386
 53.4 (cor. to 3 sig. fig.)
∴ The angle between the planes ACD and BCD is
53.4°.

(b) Let F be a point on MN such that AF  MN and


BF  MN.
AFE is a straight line.

The angle between the planes AMN and BMN is AFB.


AE 2  AB 2  BE 2 (Pyth. theorem)

AE  10 2  ( 55 ) 2 cm
 155 cm
FE  AF
1
 AE
2
155
 cm
2
Consider △BFE.
By the cosine formula, we have

249
NSS Mathematics in Action (2nd Edition) 5A Full Solutions

Consider △AFB. ∴ The angle between the path XY and the horizontal
By the cosine formula, we have ground is 13.6°.
AF 2  BF 2  AB 2
cos AFB 
2( AF )( BF )
2
 155 
   6.2249 2  102
 2 
 
 155 
2 (6.2249)
 2 
 
AFB  107 (cor. to 3 sig. fig.)
∴ The angle between the planes AMN and BMN is
107°.

20. (a)

XY  152  (30  10  10) 2 m (Pyth. theorem)


 325 m
Total distance travelled by the man
 CX  XY  YA
 (10  325  10) m
 38.0 m (cor. to 3 sig. fig.)

(b) Let G and H be the projections of X and Y on the plane


BCFE respectively.
Let Z be a point on YH at the same horizontal level as
X.

The angle between the path XY and the horizontal


ground is YXZ.
Consider △XCG.
XG
sin XCG 
XC
XG
sin 25 
10 m
XG  10 sin 25 m
Consider △YBH.
YH
sin YBH 
YB
YH
sin 25 
20 m
YH  20 sin 25 m
YZ  ( 20 sin 25  10 sin 25) m
 10 sin 25 m
Consider △XYZ.
YZ
sin YXZ 
XY
10 sin 25 m

325 m
YXZ  13.6 (cor. to 3 sig. fig.)

250
6 Applications of Trigonometry in 3-dimensional Problems

21. (a) (i) Consider △ABF. Consider △HPK.


AB By the sine formula, we have
tan AFB  HK PH
BF 
4m sin HPK sin HKP
tan 60  600 m PH
BF 
BF  2.3094 m sin (65  40) sin 50
 2.31 m (cor. to 3 sig. fig.) PH  475.8405 m
(ii) Consider △BFC.  476 m (cor. to 3 sig. fig.)
By the cosine formula, we have (ii) Consider △HPK.
By the sine formula, we have
HK PK
2
CF  BC 2
 BF 2
 2( BC ) ( BF ) cos CBF

CF 
10 2  2.3094 2
 (90  30) m

 9.0686
 2( 10)( 2.3094)
m
HPK sin
sincos KHP
 9.07 m (cor. to 3
600 m
sig. f ig. )
PK
(iii) Consider △ABF. 
sin 105 sin 25
4m
sin 60  PK  262.5160 m
AF
 263 m (cor. to 3 sig. fig.)
AF  4.6188 m
Consider △ABC.
(b) Consider △APH.
AH
AC  4 2  10 2 m (Pyth. theorem) tan APH 
PH
 116 m AH
Consider △AFC. tan 44 
By the cosine formula, we have 475.8405 m
AH  459.5138 m
AF 2  AC 2  CF 2  460 m (cor. to 3 sig. fig.)
cos FAC 
2( AF )( AC ) Consider △BPK.
BK
4.61882  ( 116 ) 2  9.06862 tan BPK 
 PK
2( 4.6188)( 116 )
BK
FAC  56.4 (cor. to 3 sig. fig.) tan 33 
262.5160 m
BK  170.4799 m
(b) Let  be the angle of elevation, where 60° <  < 90°.
 170 m (cor. to 3 sig. fig.)
4
Then BF  m.
tan 
When  increases, tan increases.
∴ BF decreases when  increases,
i.e. CF increases when  increases.
∴ William’s claim is not correct.

22. (a) (i) With the notation in the figure,

(c)

With the notation in the figure,


RB  HK  600 m
AR  ( 459.5138  170.4799) m
 289.0339 m
Consider △ABR.
AR
tan ABR 
RB
QKP  40  90  180 ( sum of △) 289.0339 m

QKP  50 600 m
ABR  25.7 (cor. to 3 sig. fig.)
QHP  65  90  180 ( sum of △) ∵ The angle of depression of B from A
QHP  25  the angle of elevation of A from B
∴ The angle of depression of B from A is 25.7°.
251
NSS Mathematics in Action (2nd Edition) 5A Full Solutions

DC
tan DAC 
23. (a) Let M be the mid-point of BC and N be the projection AC
of M on AP. hm
∴ AM  BC (prop. of isos. △) tan 60 
AC
∴ The angle that the sheet makes with the horizontal
ground is AMN.
h
AC  m
Consider △ABM. tan 60
AM 2  AB 2  BM 2 (Pyth. theorem) h
 m
AM  7 2  52 m 3
 24 m
Consider △AMN.

AN  (5  4) m  1 m
AN
sin AMN 
AM
1m

24 m
AMN  11 .7782
The angle that the sheet makes with the vertical wall
 11 .7782  90
 102 (cor. to 3 sig. fig.)

(b) In △AMN,
MN 2  AM 2  AN 2 (Pyth. theorem)

MN  ( 24 ) 2  12 m
 23 m
Area of the shadow of the sheet at noon
1
  10  23 m 2
2
 24.0 m 2 (cor. to 3 sig. fig.)

(c) The area of the shadow of the sheet is equal to the area
of △PQR, as the height and the base of the triangular
shadow do not change, even though the sun shines from
the west at a certain angle of elevation.
∴ Gloria’s claim is incorrect.

24. (a) Consider △DOC.


DC
tan DOC 
OC
hm
tan 30 
OC
h
OC  m
tan 30
 3h m
Consider △DAC.

252
6 Applications of Trigonometry in 3-dimensional Problems

Consider △COA. AB 2  AD 2  BD 2 (Pyth. theorem)


By the cosine formula, we have
AB  40  53 cm
2 2
OC 2  OA2  AC 2
cos COA   66.4003 cm
2(OC )(OA)
 66.4 cm (cor. to 3 sig. fig.)
2
 h 
( 3h)  60  2 2

 3 

2( 3h)(60)
h2
3h 2  3600 
 3
120 3h
h  1350
2

45 3h

(b) Consider △DBC.


DC
tan DBC 
BC
hm
tan 45 
BC
h
BC  m
tan 45
hm
Consider △COB.
By the cosine formula, we have
OC 2  OB 2  BC 2
cos COB 
2(OC )(OB)
( 3h) 2  902  h 2

2( 3h)(90)
2h  8100
2

180 3h
h 2  4050

90 3h

(c) ∵ COA = COB


∴ cos COA  cos COB
h 2  1350 h 2  4050

45 3h 90 3h
2h 2  2700  h 2  4050
h 2  1350
h  1350
 36.7 (cor. to 3 sig. fig.)

h 2  4050
cos COB 
90 3h
(d)
1350  4050

90 3 ( 1350 )
COB  19.5 (cor. to 3 sig. fig.)
∴ The compass bearing of B from O is N19.5°E.

25. (a) Consider △ABD.

253
NSS Mathematics in Action (2nd Edition) 5A Full Solutions

Consider △ACD.
AC 2  AD 2  CD 2 (Pyth. theorem) EA2  AD 2  DE 2  2( AD)( DE ) cos ADE
AC  40  100 cm
2 2
EA  4 2  4 2  2( 4)(4) cos 40 cm
 107.7033 cm
 2.7362 cm
 108 cm (cor. to 3 sig. fig.) Consider △CAE in Figure (b).
CE
AB  AC  BC tan CAE 
(b) (i) Let s  . EA
2 6 cm

66.4003  107.7033  141 2.7362 cm
i.e. s  2
cm
CAE  65.4854
 157.5518 cm  65.5 (cor. to 3 sig. fig.)
Area of △ABC
∴ The angle between the line AC and the
horizontal table is 65.5°.
 s ( s  AB )( s  AC )( s  BC )
157.55 18  (1 57.551 8  66.
  
(ii) Consider △CAE in Figure (b).
(157.5518 107.7033)
 (157.5518  141)
 3442.24 88 cm2
 3440 cm2 (co r. to 3
CE s ig. fig. )

(ii) Let h cm be the shortest distance between D and sin CAE 


the plane ABC. AC
BD  CD  BC 6 cm
Let t  . sin 65.4854 
2 AC
53  100  141 6
i.e. t  cm  147 cm AC  cm
2 sin 65.4854
Area of △BCD Consider △ACB in Figure (b).
By the cosine formula, we have

 s ( s  BD )( s  CD )( s  BC ) AB 2  BC 2  AC 2
cos CBA 
2( AB)( BC )
 147(147  53)(147  100)(147  141) cm 2
2
 1974 cm 2  6 
52  52   
 sin 65.4854  

1  2(5)(5)
Volume of ABCD   1974  40  cm 3
3  CBA  82.5152
Area of △ABC
 26 320 cm 3
1
Also, volume of ABCD   AB  BC  sin∠ CBA
1 2
  area of △ ABC  h cm 1 
3    5  5  sin 82.5152  cm 2
1 2 
∴ 3  3442.2488  h  26 320  12.4 cm 2 (cor. to 3 sig. fig.)
h  22.9 (cor. to 3 sig. fig.)
∴ The shortest distance between D and the 27. (a) The angle between the lines DF and BF is DFB.
plane ABC is 22.9 cm. Consider △BFG.
BF 2  BG 2  FG 2 (Pyth. theorem)
26. (a) Consider Figure (a).
AC 2  AE 2  CE 2 (Pyth. theorem) BF  6  8 cm
2 2

 10 cm
AE  (5  5) 2  6 2 cm Consider △DFE.
 8 cm DF 2  DE 2  EF 2 (Pyth. theorem)
∵ △ABD ~ △ACE (AAA)
AD AB DF  6  8 cm
2 2

  10 cm
AE AC
∴ (corr. sides, ~△s) Consider △DAB.
AD 5 cm
 DB 2  DA2  AB 2 (Pyth. theorem)
8 cm 10 cm
AD  4 cm DB  8  8 cm
2 2

 128 cm
(b) (i) The angle between the line AC and the horizontal Consider △DFB.
table is CAE. By the cosine formula, we have
Consider △DAE in Figure (b).
By the cosine formula, we have

254
6 Applications of Trigonometry in 3-dimensional Problems

BF 2  DF 2  DB 2
cos DFB 
2( BF )( DF )
10 2  102  ( 128 ) 2

2(10)(10)
DFB  68.9 (cor. to 3 sig. fig.)
∴ The angle between the lines DF and BF is 68.9°.

(b) Let N be the projection of M on the plane EFGH.


EBH  BEG (alt. s, BH // GE )
 18

ABH  180  32 (adj. s on st. line)


 148
ABE  148  18
 166
DCF  ABE
The angle between the line FM and the plane EFGH is  166
MFN. Consider △DCF.
By the cosine formula, we have
FH  EG  128 cm
1
FN  FH (property of square) DF 2  DC 2  CF 2  2( DC )(CF ) cos DCF
∴ 2
DF  7 2  82  2(7)(8) cos 166 cm
128
 cm  14.8887 cm
2
 14.9 cm (cor. to 3 sig. fig.)
Consider △MFN.
MN By the cosine formula, we have
tan MFN  CF 2  DF 2  DC 2
FN cos DFC 
6 cm 2(CF )( DF )

128 82  14.8887 2  7 2
cm 
2 2(8)(14.8887)
MFN  46.6861 DFC  6.5307
 46.7 (cor. to 3 sig. fig.) ∴ The required angle of inclination
 6.5307  18
∴ The angle between the line FM and the plane
EFGH is 46.7°.  24.5 (cor. to 3 sig. fig.)

(c) DB is the line of intersection of the planes BDF and


DE 2  EF 2  DF 2 (Pyth. theorem)
BCD.
∵ △DFB and △BCD are isosceles triangles. (b) DE  102  14.8887 2 cm
∴ FM  DB and CM  DB (prop. of isos. △)
 17.9353 cm
∴ The angle between the planes BDF and BCD is
FMC.  17.9 cm (cor. to 3 sig. fig.)
Consider △MFN. Let K be the projection of D on the horizontal table.

FMN  MFN  FNM  180 ( sum of △ )


FMN  46.6861  90  180
FMN  43.3139
CMN  90
FMC  43.3139  90
 133 (cor. to 3 sig. fig.)
∴ The angle between the planes BDF and BCD is
133°. DK  AG
 (7 sin 32  8 sin 18) cm
28. (a) With the notation in the figure,
 6.1816 cm

255
NSS Mathematics in Action (2nd Edition) 5A Full Solutions

DK 1
sin DEK  YD  BD
DE 2
6.1816 cm 1
   72 cm
17.9353 cm 2
DEK  20.2 (cor. to 3 sig. fig.) 72
∴ The angle of inclination of DE is 20.2°.  cm
2
Consider △EXQ.
EQ 2  EX 2  XQ 2 (Pyth. theorem)
2
  2

EX   9    3  cm
 2  2
 
3
 cm
2
29. (a) XZ  EZ  EX
 YD  EX
Let M be the projection of Q on CD.
∵ △ AFP △ CHQ (RHS)  72 3 
   cm
∴ FP  HQ  2 2 

∴ PE  QE
QM  ED  6 cm
Consider △PEQ.
PE 2  QE 2  PQ 2 (Pyth. theorem)
2QE  (3 cm)
2 2

9
QE  cm
2
CM  CD  MD
 9 
 6  cm
 2 

Consider △CMQ.
CQ 2  QM 2  CM 2 (Pyth. theorem)
2
 9 
CQ  62   6   cm
 2 
 
 7.1445 cm
 7.14 cm (cor. to 3 sig. fig.)

(b) (i)

Let X and Y be the mid-points of PQ and AC


respectively.
Let Z be the projection of Y on the plane EFGH.
The angle between the planes ACQP and ACD is
∠XYD, i.e. XYD   .
Consider △BCD.
BD 2  BC 2  CD 2 (Pyth. theorem)
BD  6 2  62 cm
 72 cm

256
6 Applications of Trigonometry in 3-dimensional Problems

Consider △XYZ. XY
YZ sin∠ XGY 
tan YXZ  GX
XZ
128
6 cm XY  sin 55.5501 cm
 2
 72 3
   cm  4.66 cm (cor. to 3 sig. fig.)
 2 2 
 ∴ The shortest distance between X and GH is
4.66 cm.
YXZ  65.4 (cor. to 3 sig. fig.) ∵ The radius of the sticker is greater than the shortest
XYD  YXZ (alt. s, YD // ZE ) distance between X and GH.
∴ The circular sticker of radius 5.5 cm cannot be
∴   65.4 (cor. to 3 sig. fig.)
stuck onto the inclined surface EFGH so that the
(ii) ∵ XY is a line of greatest slope of the inclined circular sticker completely lies in the region
plane ACQP. EFGH.
∴ The angle between the planes ACQP and
ACD, i.e.  is greater than the angle 31. (a) (i) Area of △BDC
between the line QC and the plane ACD. 1
∴ Tom’s claim is agreed.   BD  CD  sin∠ BDC
2
30. (a) Let N be a point on BG such that HN⊥BG. 1 
   7  7  sin 30  cm 2
GN  (18  12) cm  6 cm 2 
HN  BC  8 cm 49
Consider △GHN.  cm 2
4
GH 2  GN 2  HN 2 (Pyth. theorem)
(ii) ∵ AB  AC and BD  DC
GH  6  8 2 2
cm
∴ AD  BC (prop. of isos. △)
 10 cm
i.e. ADB  ADC  90
Consider △ABD.
(b) ∵ BG  DE and E and G are vertically above D
and B respectively.
∴ EG  DB AD 2  BD 2  AB 2 (Pyth. theorem)
Consider △BCD. AD  25 2  7 2 cm
DB 2  BC 2  CD 2 (Pyth. theorem)  24 cm
DB  8  8 cm
2 2 Volume of tetrahedron DABC
1
 128 cm   area of △ BDC  AD
3
GH  EH  EG
Let s  .  1 49 
2    24  cm 3
 3 4 
i.e. s 
10  10  128
cm  15.6569 cm  98 cm 3
2
Area of the rhombus EFGH
(b) Volume of tetrahedron DABC
 2  area of △ EGH 1
 2 s(s  GH )( s   area
EH
)( s 
of △ BDC EG  AD )
 2
15.6569 (15.6569 3  10)(15.6569  10)
(15. 6569  128 )
 93.2966 cm 2
1 1 
 93.3 cm 2
(cor.   fig.)
to 3 sig.  7  7  sin   24  cm 3
3 2 
(c) Let EG and FH intersect at X.
 196 sin  cm 3
When the centre of the circular sticker coincides with X,
the area of the sticker lying in EFGH is the maximum. When  increases from 30° to 90°, sin  increases.
Consider △EGH. ∴ When  increases from 30° to 90°, the volume of
By the cosine formula, we have the tetrahedron DABC increases.
When  increases from 90° to 150°, sin  decreases.
EG 2  GH 2  EH 2
cos∠ EGH  ∴ When  increases from 90° to 150°, the volume of
2( EG )(GH ) the tetrahedron DABC decreases.
( 128 ) 2  102  10 2 32. (a) Consider △AMD.

2( 128 )(10) 1
ADM   60  30
∠ EGH  55.5501 2
8 2
GX  cm  4 2 cm
2
Let Y be a point on GH such that XY⊥GH.
Consider △GXY.

257
NSS Mathematics in Action (2nd Edition) 5A Full Solutions

MD By the sine formula, we have


cos ADM 
AD
AB BC
MD 
cos 30  sin∠ ACB sin∠ BAC
12 cm
24 cm 13 cm
MD  12 cos 30 cm 
Let E be the projection of D on the plane ABC. sin∠ ACB sin 30
12
sin∠ ACB 
13
∠ ACB  67.3801 or 180  67.3801
 67.3801( rejected) or 112.6199
In △ABC,

Consider △DEM.
∠ ABC  ∠ ACB  ∠ BAC  180 (∠ sum of △)
DE
sin DME  ∠ ABC  180  112 .6199  30
MD
DE  37.3801
sin 30  By the cosine formula, we have
12 cos 30 cm
DE  5.1962 cm AC 2  AB 2  BC 2  2( AB )( BC ) cos∠ ABC
 5.20 cm (cor. to 3 sig. fig.)
AC  24 2  13 2  2( 24)(13) cos 37.3801 cm
∴ The height of D above the plane ABC is 5.20 cm.
 15.7846 cm
(b) The angle between AD and the plane ABC is DAE.  15.8 cm (cor. to 3 sig. fig.)
Consider △DAE.
DE (b) (i) Let M be the mid-point of BD.
sin DAE 
AD ∵ BC = DC and BM = DM
∴ CM⊥BD (prop. of isos. △)
5.1962 cm ∵ AB = AD and BM = DM

12 cm ∴ AM⊥BD (prop. of isos. △)
∵ The projection of AC on the horizontal table
DAE  25.7 (cor. to 3 sig. fig.) lies on AM.
∴ The angle between AD and the plane ABC is 25.7°. ∴ The angle between the line AC and the
horizontal table is ∠CAM.
(c) In △CMD,
30
∠BAM   15 (prop. of isos. △)
2
MC  12  (12 cos 30) cm
2 2
(Pyth. theorem) Consider △ABM.
 6 cm AM
Let F be the projection of C on the plane ABD.
cos∠ BAM 
AB
AM
cos15 
24 cm
AM  24 cos15 cm
 23.1822 cm
BM
sin∠ BAM 
The angle between BC and the plane ABD is CBF. AB
∵ MC < MD BM
∴ CF < DE sin 15 
CF DE 24 cm
∴  (∵ AD  BC ) BM  24 sin 15 cm
AD BC
i.e. sin CBF < sin DAE  6.2117 cm
∴ CBF < DAE Consider △BCM.
∴ The angle between BC and the plane ABD is less
than that in (b). BM 2  CM 2  BC 2 (Pyth. theorem)
33. (a) Join AC. CM  13 2  6.2117 2 cm
△ ABC △ ADC (SSS)
 11.4199 cm
∠BAC =∠DAC (corr. s, △ s) Consider △ACM.
60 By the cosine formula, we have
∠BAC   30
2
Consider △ABC.

258
6 Applications of Trigonometry in 3-dimensional Problems

AM 2  AC 2  CM 2 Volume of the tetrahedron CABD


cos∠ CAM 
2( AM )( AC )
1
23.1822 2  15.7846 2  11 .4199 2   area of △ ABD  CN
 3
2(23.1822)(15.7846) 1 1
∠ CAM  26.2883    24  24  sin 30  6.9908 cm 3
3 2
 26.3 (cor. to 3 sig. fig.)  336 cm 3 (cor. to 3 sig. fig.)
∴ The angle between the line AC and the
horizontal table is 26.3°.
34. (a) (i) Consider △ADF.
(ii) Height of tetrahedron CABD
 AC sin CAM AD
tan∠ AFD 
 15.7846 sin 26.2883 cm AF
12 m
 6.9908 cm tan 40 
AF
12
AF  m
tan 40
 14.3010 m
 14.3 m (cor. to 3 sig. fig.)
Consider △BCG.
BC
tan∠ BGC 
BG
5m
tan 40 
BG
5
BG  m
tan 40
 5.9588 m
 5.96 m (cor. to 3 sig. fig.)
(ii) Let N be a point on AF such that BN⊥AF.
The height of trapezium ABGF is BN.
Consider △ABN.
BN
sin ∠ BAN 
AB
BN
sin(90  50) 
30 m
BN  30 sin 40 m
 19.2836 m
Area of the shadow ABGF

1
  ( BG  AF )  BN
2
1  5 12 
     30 sin 40 m 2
2  tan 40 tan 40 
 195 m 2 (cor. to 3 sig. fig.)

(b) Similar to (a)(ii), when the sun shines from N50°W


with an angle of elevation 60°,
area of the shadow ABGF

1  5 12 
     30 sin 40 m 2
2  tan 60 tan 60 
 94.6 m 2 (cor. to 3 sig. fig.)
 195 m 2
∴ Philip’s claim is disagreed.

(c) When the sun shines from NW with an angle of


259
NSS Mathematics in Action (2nd Edition) 5A Full Solutions

elevation 40°, (ii) Consider △ACP.


consider △ABN. AP
tan∠ ACP 
AC
BN 4m
sin∠ BAN  
AB 7m
BN  30 sin(90   ) m
∠ ACP  29.7 (cor. to 3 sig. fig.)
 30 sin(90  50) m ( 0    50) ∴ The angle of elevation of P from the cat at C
 30 sin 40 m is 29.7°.
(iii) Let X be the position of the cat.
Similar to (a)(ii), for 0    50 ,
When the cat moves from B to C along the circular
area of the shadow ABGF when the sun shines from path, AX is the longest when AX is a diameter of
N W the circle.
i.e. The angle of elevation of P from X is the
smallest when AX is a diameter of the circle.
1  5 12 
     30 sin(90   ) m 2 Let r m be the radius of the circle and O be the
2  tan 40 tan 40  centre of the circle.
1  5 12 
     30 sin 40 m 2
2  tan 40 tan 40 
= area of the shadow ABGF when the sun shines from
N50W
i.e. the area of the shadow of the wall on the
horizontal ground is greater than the area obtained
in (a).
∴ Sam’s claim is agreed.

35. (a) (i) Join AC.


Consider △ACD.
By the cosine formula, we have

AC 2  AD 2  CD 2  2( AD )(CD ) cos∠ ADC


AC  3 2  5 2  2(3)(5) cos120 m
7m
(ii) Let AB = x m.
∠ ABC ∠ ADC  180 (opp. s,

∠ ABC  120  180
∠ ABC  60
cyclic quad.)

Consider △ABC.
By the cosine formula, we have

AC 2  AB 2  BC 2  2( AB )( BC ) cos∠ ABC
7 2  x 2  6 2  2( x )(6) cos 60
0  x 2  6 x  13
 ( 6) 
( 6) 2  4(1)(13)
x
2(1)
 7.6904 or  1.6904 (rejected)

AB  7.69 m (cor. to 3 sig. fig.)

(b) (i) Consider △ABP.


AP
tan∠ ABP 
AB
4m

7.6904 m
∠ ABP  27.5 (cor. to 3 sig. fig.)
∴ The angle of elevation of P from the cat at B
is 27.5°.

260
6 Applications of Trigonometry in 3-dimensional Problems

∠AOC = 2 × 60° = 120° (∠ at centre twice ∠ 4. Answer: B


at ⊙ce) Let a be the length of the side of square ABCD.
Consider △OAC. Consider △DCB.
By the cosine formula, we have
DB  a2  a2 (Pyth. theorem)
 2a
AC 2  OA 2  OC 2  2(OA)(OC ) cos∠ AOC
Consider △CBE.
7 2  r 2  r 2  2( r )( r ) cos120 CE
sin 30 
49  3r 2
a
r  4.0415 or  4.0415 (rejected) a
CE 
When AX is a diameter of the circle, 2
AP Consider △DBF.
tan∠ AXP 
AX a


4m sin DBF  2
4.0415  2 m 2a
∠ AXP  26.3 (cor. to 3 sig. fig.) DBF  20.7 (cor. to 3 sig. fig.)
∴ The angle of elevation of P from the cat first
decreases from 27.5° to 26.3°, then increases 5. Answer: A
to 29.7° when the cat moves from B to C Let N be the mid-point of DE.
along the circular path.

Multiple Choice Questions (p. 6.62)


1. Answer: C
FG is the line of intersection of the planes AFGD and
EFGH.
∵ AF  FG and EF  FG, DG  FG and HG  FG
∴ The angles between the planes AFGD and EFGH are
AFE or DGH.
∴ The answer is C.
N is the projection of M on the plane CHED.
2. Answer: A
The angle between the line MH and the plane CHED is
∵ The plane EMF is a part of the plane ABFE.
∴ The angle between the planes EMF and ABCD is the MHN.
angle between the planes ABFE and ABCD. 10
∵ The angle between the planes ABFE and ABCD is NE  cm
2
∠CBF.
∴ The angle between the planes EMF and ABCD is  5 cm
∠CBF. Consider △NHE.
HN 2  HE 2  NE 2 (Pyth. theorem)
3. Answer: D
HN  4 5
2 2
cm
BD  k 2  k 2 cm (Pyth. theorem)
 2 k cm  41 cm
Consider △MHN.
HB  ( 2 k ) 2  k 2 cm (Pyth. theorem)
 3k cm MN
tan MHN 
Let M be the point of intersection of AG and BH. HN
∵ AG  HB  3k cm 4 cm

3k 41 cm
∴ MA  MB  cm
2 MHN  32 (cor. to the nearest degree)
Consider △MAB.
∴ The angle between the line MH and the plane CHED is
By the cosine formula, we have
2 2
32.
 3k   
    3k   k 2 6. Answer: A
 2   2 
cos      Let J be the mid-point of BC.
 3k   3k 
2  
 2  2 
  
1

3
∴ The answer is D.

The angle between the planes HBC and ABCD is HJK.

261
NSS Mathematics in Action (2nd Edition) 5A Full Solutions

4
AK  cm
2
 2 cm
Consider △HAK.
HK 2  HA2  AK 2 (Pyth. theorem)
HK  3 2
2 2
cm
With the notations in the figure, let w m and h m be the
 5 cm width and the height of the wall respectively.
Consider △HJK. A  wh
Consider △ABE.
HK AE
tan HJK  tan∠ ABE 
JK AB
5 cm hm
 tan  
4 cm AB
HJK  29 (cor. to the nearest degree) h
AB  m
∴ The angle between the planes HBC and ABCD is 29. tan 
∠BAZ =∠XAY = (vert. opp.∠s)
7. Answer: C Area of ABCD
Total surface area of a regular tetrahedron
1
 4  area of each triangle  2  AB  AD  sin∠ BAD
2
 1  h
  4   10  10  sin 60  cm 2   w  sin(90   ) m 2
 2  tan 
 100 3 cm 2 wh cos
 m2
tan 
8. Answer: D
A cos
Let AB = h.  m2
Consider △ABC. tan 
AB
tan∠ ACB 
BC 10. Answer: A
With the notation in the figure,
h
tan 25 
BC
h
BC 
tan 25
Consider △ABD.
AB
tan∠ ADB 
BD
h
tan 50  Consider △ABQ.
BD 2 km
h tan 20 
BD  QB
tan 50 2
Consider △BCD. QB  km
tan 20
BC
tan∠ BDC  Consider △ABP.
BD 2 km
h tan 25 
PB
 tan 25 2
h PB  km
tan 25
tan 50 Consider △PQB.
tan 50 By the cosine formula, we have
 2 2
tan 25  2   2 
82     
∠ BDC  69 (cor. to the nearest degree)  tan 20    tan 25 
cos PQB 
 2 
9. Answer: C 2(8) 
 tan 20 
PQB  30.4 (cor. to 3 sig. fig.)
∴ The bearing of A from Q is S30.4E.

262
6 Applications of Trigonometry in 3-dimensional Problems

In △PRC,
11. Answer: B
PR  32  32 cm (Pyth. theorem)
2
CQ  6  cm  4 cm  18 cm
3 In △QRC,
1
PC  CR  6  cm  3 cm QR  42  32 cm (Pyth. theorem)
2
In △PQC,  5 cm
Consider △PQR.
PQ  32  4 2 cm (Pyth. theorem) PQ  QR  PR
Let s  .
 5 cm 2
55 18  18 
i.e. s cm   5   cm
2  2 
 
Area of the base PQR

 s( s  PQ )( s  QR )( s  PR )

 18   18 
 5     5   5
 2   2 
   

 18 
 5   18 
 2 
 
2
 9.6 cm (cor. to 1 d.p.)

12. Answer: B
Consider △ABD.
∵ AB = AD and BP = DP
∴ AP⊥BD and ∠BAP =∠DAP (prop. of isos. △)
Consider △ABP.
90
∠BAP   45
2
AP
cos∠ BAP 
AB
AP
cos 45 
2 cm
AP  2 cos 45 cm
 2 cm
Similarly, AQ  2 cm
Consider △ABC.
By the cosine formula, we have
BC 2  AB 2  AC 2  2( AB )( AC ) cos∠ BAC
BC  2 2  2 2  2( 2)( 2) cos 30 cm

 8  4 3 cm

 2 2 3 cm
∵ P and Q are mid-points of BD and CD respectively.
BC
PQ 
2
∴ (mid-pt. theorem)
2 2 3
 cm
2
 2  3 cm

263
NSS Mathematics in Action (2nd Edition) 5A Full Solutions

Consider △APQ. XT
By the cosine formula, we have cos AXT 
AX
AP 2  AQ 2  PQ 2
cos∠ PAQ  XT  AX cos 70
2( AP )( AQ )
 (25  CX ) cos 70
( 2 )2  ( 2 )2  ( 2  3 )2  3.9785 cm

2( 2 )( 2 )
∠ PAQ  21 (cor. to the nearest degree)

Exam Focus

Exam-type Questions (p. 6.66)


1. (refer to Amendment List in Sept 2016)
(a) (i) Consider △ABC.
By the cosine formula, we have
AC 2  BC 2  AB 2
cos∠ ACB 
2( AC )( BC )
252  152  202

2(25)(15)
∠ ACB  53.1301
 53.1 (cor. to 3 sig. fig.)
∴ ∠ BCX  ∠ ACB  53.1
(ii) In △BCX,

∠ BCX  ∠ BXC  ∠ CBX  180 ( sum of △)


∠ CBX  180  53.1301  70
 56.8699
By the sine formula, we have
CX BC

sin∠ CBX sin∠ BXC
15 sin 56.8699
CX  cm
sin 70
 13.3676 cm
 13.4 cm (cor. to 3 sig. fig.)

(b) (i) Suppose BX is produced to a point T such that


AT  CT and AT  BT .

Consider △ATX.
AXT  180  AXB (adj. s on st.

 180  110 
 70
line)

AT
sin AXT 
AX
AT  AX sin 70
 (25  CX ) sin 70
 10.9308 cm

264
6 Applications of Trigonometry in 3-dimensional Problems

Consider △CXT. BQ
CXT  180  BXC (adj. s on st. sin∠ BCQ 
∠ CQ
 180  70
BQ
 110  sin 55 
line)
15 m
BQ  15 sin 55 m
By the cosine formula, we have  12.2873 m
CT  CX 2
 XT 2
 2(CX )( XT ) cos C

13.3676  3.9785 2 2
 cm
 2(13.3676)(3.9785) cos110 
 15.1954 cm

Consider △ACT.

AT 2
 CT 2  AC 2 (Pyth. theorem)
AC  10.93082  15.19542 cm
 18.7185 cm
 18.7 cm (cor. to 3 sig. fig.)
(ii) The tetrahedron ABXC has a maximum height
when △AXB is perpendicular to the horizontal
plane.
i.e. The tetrahedron ABXC has a maximum
volume when △AXB is perpendicular to the
horizontal plane.
∴ When AC increases from 15 cm to 18.7 cm,
the volume increases; when AC increases
from 18.7 cm to 25 cm, the volume decreases.

2. (a) (i) Consider △CPQ.


By the cosine formula, we have

PQ 2  CP 2  CQ 2  2(CP )(CQ ) cos∠ PCQ


PQ  16 2  152  2(16)(15) cos 80 m
 19.9411 m
 19.9 m (cor. to 3 sig. fig.)
(ii) Consider △ACP.
AC
cos∠ ACP 
CP
AC
cos 25 
16 m
AC  16 cos 25 m
 14.5009 m
AP
sin∠ ACP 
CP
AP
sin 25 
16 m
AP  16 sin 25 m
 6.7619 m
Consider △BCQ.
BC
cos∠ BCQ 
CQ
BC
cos 55 
15 m
BC  15 cos 55 m
 8.6036 m

265
NSS Mathematics in Action (2nd Edition) 5A Full Solutions

Let N be a point on BQ such that PN⊥BQ.


NQ  BQ  AP
 (12.2873  6.7619) m
 5.5254 m
Consider △NPQ.

NP 2  NQ 2  PQ 2 (Pyth. theorem)
NP  19.9411 2  5.5254 2 m
 19.1603 m
AB  NP  19.1603 m
Consider △ABC.
By the cosine formula, we have
AC 2  BC 2  AB 2
cos∠ ACB 
2( AC )( BC )
14.5009 2  8.6036 2  19.16032

2(14.5009)(8.6036)
∠ ACB  109.3850
 109 (cor. to 3 sig. fig.)

(b)

∠ ACD  ∠ ACB  180 (adj. s on st. line)


∠ ACD  180  109.3850
 70.6150
Consider △ACD.
By the cosine formula, we have
AD 2  AC 2
 DC 2
 2( A C )( DC ) cos∠ AC D
14.5009 2  8.6036 2
AD  m
 2(14.5009)(8.6036) cos 70.6150
 14.1943 m
Consider △ADP.
DP 2  AP 2  AD 2 (Pyth. theorem)
DP  6.7619  14.19432 m
2

 15.7 m (cor. to 3 sig. fig.)


∴ DP is shorter than CP.
∴ Simon’s claim is disagreed.

3. (a) Let M be the mid-point of FD.


∵ AF = AD and EF = ED with DM = FM
∴ AM⊥DF and EM⊥DF (prop. of isos. △)
∵ DF is the line of intersection of the planes AFD
and EFD with AM⊥DF and EM⊥DF.
∴ The angle between the planes AFD and EFD is
∠AME.
Consider △EFM.
EM 2  FM 2  EF 2 (Pyth. theorem)
2
 12 
EM  20 2    cm
 2 
 364 cm
Consider △AEM.
AE
tan∠ AME 
EM
AE
tan 60 
364 cm
AE  364 tan 60 cm
 33.0454 cm
 33.0 cm (cor. to 3 sig. fig.)

266
6 Applications of Trigonometry in 3-dimensional Problems

(b) (i) Distance between XY and BC BG


EM cos∠ BGC 
 CG
2 h
364 cos 20 
 cm CG
2 h
33.0454 cm CG 
tan   cos 20
364
cm
2
  73.9 (cor. to 3 sig. fig.)
(ii) ∵ XF is not a line of greatest slope on the plane
XFDY.
∴ The angle  between the planes XFDY and
EFD must be greater than that between the
line XF and the plane EFD.

4. Answer: C

5. Answer: B
Let M be the mid-point of BC and N be the projection of V
on the plane ABCD.
MN = 2 cm
Consider △VMN.
MN
tan∠ MVN 
VN
 2 cm
tan 
2 VN
2
VN  cm

tan
2
∵ Volume of the pyramid = 16 cm3
1 2
 42   16
3 
tan
∴ 2
 2
tan 
2 3

6. Answer: A
Let BG = h.
Consider △ABG.
BG
cos∠ AGB 
AG
h
cos 30 
AG
h
AG 
cos 30
AB
tan∠ AGB 
BG
AB
tan 30 
h
AB  h tan 30
Consider △BCG.

267
NSS Mathematics in Action (2nd Edition) 5A Full Solutions

BC 8. Answer: D
tan∠ BGC  For I:
BG ∵ CF = DE and BF = AE
BC CF DE
tan 20  
h BF AE

BC  h tan 20 tan ∠ CBF  tan ∠ DAE
Consider △ABC. tan b  tan c
AC 2  AB 2  BC 2 (Pyth. theorem)
b c
AC  ( h tan 30) 2  ( h tan 20) 2 ∴ I is true.
For II:
Consider △ACG.
∵ BC is a line of greatest slope of the inclined plane
By the cosine formula, we have
ABCD and DG is not a line of greatest slope of the
c os∠ A GC 
inclined plane ABCD.
A G 2  CG 2

2( A G )(C
2
 h 
 
 c os 30  

∴ The angle between BC and the horizontal ground is





( h


2
t an

h
3 0

greater than the angle between DG and the horizontal


 co s 3
2
h
2

 c os 3 0 c


ground. c os
h
2
2

3 0
(1 

i.e. ∠ CBF ∠ DGE


2
2 h
 2
2 h
c os 2 0  co s 3

bd
 co s 2 0  c o s 3
∠ A GC  3 6 (c or. to t

∴ II is true.
7. Answer: B
For III:
Consider △ADE.
∵ BF = AE and AG < HB
AD ∴ EG < HF
tan∠ AED 
DE 1 1

AD EG HF
DE 
tan  DE

CF
Consider △AMD. ∴
EG HF
DM tan∠ DGE  tan∠ CHF
tan∠ DAM 
AD tan d  tan a
DM  AD tan 
d a
∵ DM = CM ∴ III is true.
∴ CD  2 AD tan  ∴ The answer is D.
Consider △CDE.

tan∠ CED 
CD Investigation Corner (p. 6.73)
DE
2 AD tan 
tan  
AD
tan 
 2 tan  tan 

1.

360
BEC  (s at a pt.)
3
 120
Let BE  EC  y .
Consider △BEC.
BC 2  BE 2  EC 2  2( BC )( EC ) cos BEC
a 2  y 2  y 2  2( y )( y ) cos120
a2  3 y 2
a2
y
3
1
 a
3

268
6 Applications of Trigonometry in 3-dimensional Problems

1 Consider △ABE.
∴ BE  a
3 AB 2  AE 2  BE 2 (Pyth. theorem)
2
 1 
AE  a 2   a 
 3 
2
 a
3

2. Let OA  x .
OE  AE  OA
2
 ax
3
Consider △OBE.
OB 2  OE 2  BE 2 (Pyth. theorem)
2 2
 2   1 
x2   a  x    a 
 3 
   3 
2 2 2 1
x2  a 2 ax  x 2  a 2
3 3 3
2
2 ax  a 2
3
a
x
2
2
3
1 3
 a
2 2
1 3
∴ OA  a
2 2

3. Consider △OAB.
By the cosine formula, we have
OA2  OB 2  AB 2
cos AOB 
2(OA)(OB)
2 2
1 3  1 3 
 a  a   a2
2 2  2 2 
  
 1 3  1 3 
2 a  a
 2 2  2 2 
  
1 2
a 
 4
3 2
a
4
1

3
AOB  109 (cor. to the nearest degree)
∴ The bond angle of the diamond is 109.

269

You might also like